Mains Practice Questions

Back

Total Questions: 115

Question:
What is a tsunami? How earthquake trigger a tsunami? Discuss why tsunamis keep forming in the island country.
 
 
Introduction:
 
A Simple Introduction about Tsunami

A tsunami, derived from the Japanese words "tsu" (harbor) and "nami" (wave), refers to a series of long-wavelength waves triggered by various natural phenomena, most commonly earthquakes, volcanic eruptions, or underwater landslides. These waves can propagate across vast distances in the ocean, causing devastation upon reaching coastal areas.

 
Body:
 
It is the central part of the answer and one should understand the demand of the question to provide rich content.
  • Tsunamis are primarily triggered by earthquakes that involve the sudden vertical movement of the ocean floor. When an earthquake occurs beneath the ocean, particularly at subduction zones where tectonic plates converge, the displacement of large volumes of water leads to the formation of tsunami waves. This displacement can result from the upward or downward movement of the sea floor, generating powerful waves that propagate outward from the earthquake epicenter.
  • The phenomenon of tsunamis occurring frequently in island countries, such as Japan and those situated in the Pacific Ocean, can be attributed to their location within seismically active regions. These regions, notably the Pacific Ring of Fire, are characterized by the convergence of multiple tectonic plates, leading to frequent seismic activity, volcanic eruptions, and subsequent tsunami formation.
  • For instance, Japan, situated along the Pacific Ring of Fire, experiences a high frequency of earthquakes and volcanic eruptions due to the complex interaction of tectonic plates. In 2011, Japan witnessed the devastating impact of a 9.0 magnitude earthquake and ensuing tsunami, resulting in widespread destruction and loss of life.
  • Similarly, India's coastal regions, particularly Gujarat, are vulnerable to tsunamis resulting from seismic activity in the Mekran coast. The Indian Ocean region, including countries like Indonesia, India, and Sri Lanka, has also experienced catastrophic tsunamis, such as the 2004 Indian Ocean tsunami triggered by a 9.3 magnitude earthquake.
 
Conclusion: 
 
The ending of the answer should be on a positive note and it should have a forward-looking approach.

While not all earthquakes or volcanic eruptions lead to tsunami formation, the propensity for tsunamis in island countries and coastal regions is heightened due to their exposure to seismic activity and geological instability. Understanding the underlying mechanisms of tsunami generation and implementing effective early warning systems are crucial steps in mitigating the impact of these natural disasters and safeguarding vulnerable coastal communities.

 
 
Other Points to Ponder 

Pacific Island Countries

Tsunami in India

 

 

Previous Year Questions

1. Discuss about the vulnerability of India to earthquake related hazards. Give examples including the salient features of major disasters caused by earthquakes in different parts of India during the last three decades. (2021)

2. On December 2004, tsunami brought havoc on fourteen countries including India. Discuss the factors responsible for occurrence of tsunami and its effects on life and economy. In the light of guidelines of NDMA (2010) describe the mechanisms for preparedness to reduce the risk during such events. (2017)

 

(mains - general-studies-3 ) 05-Apr 2024
Question:
What is India’s first polarimetry mission X-ray Polarimeter Satellite (XPoSat)? Discuss its significance.
 
 
Introduction:
 
A Simple Introduction about India's first polarimetry mission

India's first polarimetry mission, the X-ray Polarimeter Satellite (XPoSat), marks a significant milestone in the country's space exploration endeavours. Designed to delve into the intricate dynamics of bright astronomical X-ray sources, XPoSat stands as a testament to India's growing prowess in space science and technology. With its unique capabilities and cutting-edge payloads, this mission promises to unravel the mysteries of the universe like never before.

 
Body:
 
It is the central part of the answer and one should understand the demand of the question to provide rich content.
  • XPoSat, positioned in low Earth orbit at approximately 650 km with a low inclination of around 6 degrees, boasts two state-of-the-art scientific payloads: the Indian X-ray Polarimeter (POLIX) and the X-ray Spectroscopy and Timing (XSPECT). Developed by esteemed institutions such as the Raman Research Institute and the UR Rao Satellite Centre in Bengaluru, these payloads are at the forefront of X-ray polarimetry research.
  • The POLIX instrument, a pioneering creation, operates within the medium X-ray energy band of 8 to 30 kilo electron Volts (keV), making it the world's first instrument of its kind. Meanwhile, the XSPECT payload is geared towards conducting fast timing and high spectroscopic resolution in the soft X-ray energy band ranging from 0.8 to 15 keV. Together, these payloads empower XPoSat to explore a wide range of astronomical phenomena with unprecedented precision and depth.
  • The significance of the XPoSat mission lies in its transformative potential for X-ray astronomy. By enabling X-ray polarisation measurements from bright sources in the medium energy band, XPoSat opens new avenues for scientific inquiry and discovery. From persistent sources like targeted and known astronomical objects to transient sources such as pulsars, active galactic nuclei, and magnetars, XPoSat's observations promise to revolutionize our understanding of the cosmos.
  • X-rays, emitted by various celestial bodies under extreme conditions, can become polarized due to interactions with surrounding materials or intense magnetic fields. Through the analysis of polarised X-rays, XPoSat aims to unravel the secrets of phenomena like black holes, neutron stars, and their surroundings, shedding light on some of the most enigmatic aspects of the universe.
 
Conclusion: 
 
The ending of the answer should be on a positive note and it should have a forward-looking approach.

With an estimated mission life of five years, XPoSat is poised to make groundbreaking discoveries during its operational tenure. By observing polarised X-rays from a diverse array of celestial sources, this mission will contribute significantly to humanity's quest for knowledge about the cosmos. As XPoSat embarks on its journey of exploration, it reaffirms India's commitment to advancing space science and unlocking the mysteries of the universe for the betterment of humankind.

 
Other Points to Ponder 

Other payloads of the Mission

POLIX

XSPECT

 

Previous Year Questions

1. Launched on 25th December, 2021, James Webb Space Telescope has been much in the news since then. What are its unique features which make it superior to its predecessor Space Telescopes? What are the key goals of this mission? What potential benefits does it hold for the human race? (2022)

2. India has achieved remarkable successes in unmanned space missions including the Chandrayaan and Mars Orbiter Mission, but has not ventured into manned space mission. What are the main obstacles to launching a manned space mission, both in terms of technology and logistics? Examine critically. (2017)

 

(mains - general-studies-3 ) 05-Apr 2024
Question:
What are the first advance estimates? How can GDP be estimated by using first advance estimates?
 
 
Introduction:
 
A Simple Introduction about the First Advance Estimates 

The First Advance Estimates (FAEs) are preliminary projections of economic growth presented by the Ministry of Statistics and Programme Implementation (MoSPI) in early January each year. These estimates provide an initial assessment of the country's Gross Domestic Product (GDP) for the ongoing financial year.

Body:
 
It is the central part of the answer and one should understand the demand of the question to provide rich content.

Methodology of Estimation

The FAEs are formulated using the benchmark-indicator method, which involves extrapolating the previous year's GDP data based on relevant indicators reflecting sectoral performance. These indicators include data on industrial production, agricultural output, trade, services, and other economic activities.

Significance of FAEs

  • The FAEs serve as crucial inputs for policymakers, economists, and stakeholders, offering insights into the trajectory of the economy and guiding decision-making processes.
  • They are the last GDP data released before the presentation of the Union Budget, providing policymakers with important information to formulate fiscal policies and budgetary allocations.

Key Findings of FAEs

  • The FAEs for the current fiscal year (2023-24) suggest a GDP growth rate of 7.3%, indicating a robust economic performance.
  • This growth rate surpasses earlier expectations, with projections initially hovering between 5.5% and 6.5%. The upward revision underscores the resilience and strength of India's economic recovery.
  • Components contributing to GDP growth include Private Final Consumption Expenditure (PFCE), Gross Fixed Capital Formation (GFCF), Government Final Consumption Expenditure (GFCE), net exports, private consumption demand, and government spending.
Conclusion: 
 
 The ending of the answer should be on a positive note and it should have a forward-looking approach.

The release of First Advance Estimates marks a significant milestone in assessing the economic outlook for the ongoing financial year. These estimates offer valuable insights into the performance of various sectors and provide policymakers with essential information to formulate appropriate strategies for sustainable economic growth. As India strives to navigate through economic challenges and capitalize on emerging opportunities, the FAEs play a pivotal role in guiding decision-making and shaping policy interventions for fostering inclusive and resilient economic development.

 
Other Points to Ponder 

National Statistical Office

Per Capita Income

Real GDP and Nominal GDP

 

 

Previous Year Questions

1. Faster economic growth requires increased share of the manufacturing sector in GDP, particularly of MSMEs. Comment on the present policies of the Government in this regard (2023)

2. Explain the difference between computing methodology of India’s Gross Domestic Product (GDP) before the year 2015 and after the year 2015. (2021)

 

(mains - general-studies-3 ) 05-Apr 2024
Question:
What is cyber kidnapping? Discuss the major cybercrimes reported in India in 2022, according to data released by the National Crime Records Bureau.
 
 
Introduction:
 
A Simple Introduction about Cyber Kidnapping

Cyber kidnapping is a form of crime where perpetrators manipulate victims into believing that a loved one has been abducted and demand ransom for their release. This nefarious tactic involves coercing victims to hide while contacting their family or friends for ransom. Perpetrators may use fabricated evidence, such as manipulated images or threatening messages, to convince victims of the authenticity of the kidnapping.

 
Body:
 
It is the central part of the answer and one should understand the demand of the question to provide rich content.

Cybercrimes in India

  • According to the National Crime Records Bureau (NCRB), cybercrimes in India witnessed a significant increase in 2022, with a 24% rise compared to the previous year.
  • The NCRB report 'Crime in India' highlighted that out of 65,893 cybercrime cases registered, fraud accounted for 64.8% of the cases, followed by extortion (5.5%) and sexual exploitation (5.2%).
  • Economic offences, including forgery, cheating, and fraud (FCF), constituted a significant portion of cybercrimes, with 1,70,901 cases reported in the previous year.
  • Crimes against women also saw a rise, with 4,45,256 cases registered in 2022, showing a 4% increase compared to 2021. These included cases of cruelty by husband or relatives, kidnapping, abduction, assault, and rape.

Impact of Cyber Kidnapping

  • Cyber kidnapping not only inflicts emotional distress on victims and their families but also poses significant financial risks. Victims may succumb to the pressure of paying ransom out of fear for their loved one's safety, leading to financial loss.
  • The psychological trauma experienced by victims can have long-lasting effects, including anxiety, depression, and post-traumatic stress disorder (PTSD).
  • Moreover, cyber kidnapping undermines trust in online communication channels and creates a sense of insecurity among internet users.

Preventive Measures

  • To mitigate the risk of falling victim to cyber kidnapping, individuals should exercise caution while sharing personal information online, especially on social media platforms.
  • It is essential to verify the authenticity of any distress calls or messages received and refrain from acting impulsively under pressure.
  • Maintaining strong cybersecurity practices, such as using secure passwords, updating security software, and avoiding suspicious links or emails, can help prevent cybercrimes.
 
Conclusion: 
 
The ending of the answer should be on a positive note and it should have a forward-looking approach.

Cyber kidnapping represents a grave threat in the digital age, exploiting the vulnerabilities of individuals and exploiting their fears for financial gain. As cybercrimes continue to evolve, law enforcement agencies and cybersecurity experts must collaborate in combatting these threats and safeguarding individuals' online safety and security. Additionally, raising awareness about the tactics employed by cybercriminals and promoting responsible online behaviour can empower individuals to protect themselves and their loved ones from falling victim to cyber kidnapping and other forms of online extortion.

 
 
Other Points to Ponder 

Artificial intelligence

Cognizable and Uncognizable crimes

 

Previous Year Questions

1. What are the different elements of cyber security? Keeping in view the challenges in cyber security, examine the extent to which India has successfully developed a comprehensive National Cyber Security Strategy. (2022)

2. Discuss different types of cyber crimes and measures required to be taken to fight the menace. (2020)

3. Discuss the potential threats of Cyber attack and the security framework to prevent it. (2017)

 

(mains - general-studies-3 ) 05-Apr 2024
Question:
Discuss how megacities are tackling air pollution? Highlight the impact of air pollution on economic growth.
 
 
Introduction:
 
A Simple Introduction about Air Pollution
 
Air pollution refers to the presence of harmful or excessive quantities of substances in the air, which can pose risks to human health, the environment, and ecosystems. These substances, known as pollutants, can be natural or man-made and include gases, particulate matter, volatile organic compounds, and biological molecules. Air pollution can arise from various sources, including industrial activities, transportation, agriculture, energy production, and natural phenomena such as volcanic eruptions and wildfires.
 
Body:
 
It is the central part of the answer and one should understand the demand of the question to provide rich content.

Air pollution is a global health crisis, causing approximately seven million deaths worldwide each year according to the World Health Organisation (WHO). Megacities, facing significant pollution challenges, are implementing various strategies to tackle this issue.

Strategies for Tackling Air Pollution:

  • Clean Transport: Megacities are promoting cleaner and fewer automobiles, encouraging walking, cycling, and improving public transport to reduce vehicular emissions.
  • Industrial Sources: Cities like Delhi are focusing on reducing dust from construction sites and transitioning to cleaner fuels and more efficient techniques to mitigate industrial emissions.
  • Waste Management: Effective waste management is essential to prevent open burning, a significant contributor to air pollution. Regulation and infrastructure improvements are crucial components of city plans against pollution.
  • Clean Energy: Cities worldwide, including those in China, are making strides in lowering pollution levels by transitioning to cleaner energy sources.

Impact of Air Pollution on Economic Growth:

  • GDP Risk: The Reserve Bank of India's Department of Economic and Policy Research (DEPR) reports that up to 4.5% of India's GDP could be at risk by 2030 due to lost labour hours from climate change issues, including extreme heat and humidity.
  • Economic Losses: Studies have shown large inter-state variations in economic losses due to air pollution, with the biggest impacts seen in states with low per-capita GDP such as Uttar Pradesh, Bihar, Rajasthan, Madhya Pradesh, and Chhattisgarh.
  • Factors Contributing to Losses: Economic losses from air pollution manifest in lower labour productivity, consumer footfall, asset productivity, increased health expenses, welfare losses, and premature mortality.
  • City-Specific Impact: Delhi has been identified as the city with the highest per-capita economic loss due to pollution among major Indian cities, highlighting the severity of the issue.
 
Conclusion: 
 
 The ending of the answer should be on a positive note and it should have a forward-looking approach.

Addressing air pollution is crucial for economic growth, as it impacts productivity, healthcare costs, and overall welfare. Urgent measures are needed in megacities, where pollution levels are alarmingly high, to mitigate the adverse effects and pave the way for sustainable development.

 
Other Points to Ponder 

AQI

Pollution in Northern India

 

Previous Year Questions

1. Describe the key points of the revised Global Air Quality Guidelines (AQGs) recently released by the World Health Organisation (WHO). How are these different from its last update in 2005? What changes in India’s National Clean Air Programme are required to achieve revised standards? (2021)

2. What are the key features of the National Clean Air Programme (NCAP) initiated by the government of India? (2020)

 

(mains - general-studies-3 ) 05-Apr 2024
Question:
 How was hybridisation achieved in mustard? Discuss concerns associated with the crop.
 
 
Introduction:
 
A Simple Introduction about Hybridisation

Hybridisation, the process of crossing genetically dissimilar plant varieties to produce first-generation (F1) offspring with higher yields, is challenging in mustard due to its self-pollinating nature. However, scientists have achieved hybridisation in mustard through genetic modification (GM), specifically with the development of hybrid mustard DMH-11 by the Centre for Genetic Manipulation of Crop Plants (CGMCP) at Delhi University. This hybrid mustard contains two alien genes from Bacillus amyloliquefaciens, a soil bacteria.

 
Body:
 
It is the central part of the answer and one should understand the demand of the question to provide rich content.

Genetic Modification Process:

  • The first gene, called 'barnase', induces male sterility by impairing pollen production in the mustard plant.
  • The male-sterile plant is then crossed with a fertile parental line containing the second gene, 'barstar', which counteracts the effects of the barnase gene.
  • The resulting F1 progeny exhibits high yield and retains the ability to produce seeds.

Concerns Associated with the Crop:

  • The development and cultivation of genetically modified (GM) crops, such as hybrid mustard DMH-11, raise concerns related to the precautionary principle.
  • This principle suggests that in the absence of scientific consensus or adequate information, caution should be exercised when dealing with innovations that could have significant adverse impacts on human health or the environment.
  • Decision-making surrounding GM crops often faces criticism for relying solely on the precautionary principle, which some argue impedes scientific progress or leads to inaction.
 
Conclusion: 
 
 The ending of the answer should be on a positive note and it should have a forward-looking approach.

In India, the Genetic Engineering Appraisal Committee (GEAC), operating under the Ministry of Environment, Forest and Climate Change (MoEF&CC), is responsible for evaluating proposals related to the release of GM organisms into the environment. While Bt cotton is the only GM crop currently permitted in India, the cultivation of genetically modified varieties of edible oils and other crops is prevalent in the country. As discussions continue on the safety and implications of GM crops, regulatory bodies like the GEAC play a crucial role in assessing their environmental and human health impacts.

 
Other Points to Ponder 

Bt Cotton

Why is the Supreme Court (SC) examining the matter of GM crops?

 

Previous Year Questions

1. What is Integrated Farming System? How is it helpful to small and marginal farmers in India? (2022)

2. How has the emphasis on certain crops brought about changes in cropping patterns in recent past? Elaborate the emphasis on millets production and consumption. (2018)

 

(mains - general-studies-3 ) 05-Apr 2024
Question:
What is ‘Pradhan Mantri Suryodaya Yojana’? Discuss why expansion of solar energy is important for India.
 
 
Introduction:
 
A Simple Introduction about Pradhan Mantri Suryodaya Yojana

The 'Pradhan Mantri Suryodaya Yojana' is a scheme aimed at installing solar power systems on rooftops for residential consumers in India. By reducing electricity bills for both the poor and middle class, the scheme aligns with India's objective of achieving self-reliance in the energy sector.

 
Body:
 
It is the central part of the answer and one should understand the demand of the question to provide rich content.

India's Solar Energy Capacity:

India has made significant strides in solar energy, with an installed capacity of approximately 73.31 GW of solar power as of December 2023. However, rooftop solar capacity remains relatively modest, estimated at around 11.08 GW by December 2023. States like Rajasthan and Gujarat lead in total solar capacity, while Gujarat and Maharashtra top the list in rooftop solar capacity.

Importance of Solar Energy Expansion:

Expanding solar energy is crucial for India due to its rapidly increasing energy demand, projected to be the highest globally in the next three decades. To meet this demand sustainably, India needs reliable and clean energy sources beyond traditional coal plants. Therefore, the country aims to achieve 500 GW of renewable energy capacity by 2030, emphasizing the importance of solar energy expansion.

Rooftop Solar Programme:

The Rooftop Solar Programme, launched in 2014, aims to boost rooftop solar capacity in the residential sector by providing financial assistance and incentives to distribution companies (DISCOMs). The programme's target is to increase rooftop solar capacity to 40 GW by March 2026, with ongoing efforts in its second phase.

 
Conclusion: 
 
 The ending of the answer should be on a positive note and it should have a forward-looking approach.

In addition to reducing electricity bills for consumers, surplus solar power generated from rooftop installations can be exported to the grid, providing monetary benefits to consumers. As India continues to focus on expanding solar energy capacity, initiatives like the Pradhan Mantri Suryodaya Yojana play a pivotal role in achieving the country's renewable energy goals and fostering sustainability in the energy sector.

 
Other Points to Ponder 

Other Renewable Energy

International Solar Alliance (ISA)

 

Previous Year Questions

1. Explain the purpose of the Green Grid Initiative launched at World Leaders Summit of the COP26 UN Climate Change Conference in Glasgow in November, 2021. When was this idea first floated in the International Solar Alliance (ISA)? (2021)

2. Describe the major outcomes of the 26th session of the Conference of the Parties (COP) to the United Nations Framework Convention on Climate Change (UNFCCC). What are the commitments made by India in this conference? (2021)

 

(mains - general-studies-3 ) 05-Apr 2024
Question:
How does Artificial intelligence (AI) help in health sector. Give examples in support of your answer.
 
 
Introduction:
 
A Simple Introduction about Artificial intelligence

Artificial intelligence (AI) is revolutionizing the healthcare sector, providing innovative solutions for diagnosis, treatment, and prevention. Mumbai’s Tata Memorial Hospital (TMH), India’s premier cancer hospital, exemplifies this trend by leveraging AI to enhance cancer detection through its 'Bio-Imaging Bank.' This tailored algorithm aids in early-stage cancer identification, showcasing the transformative potential of AI in healthcare.

 
Body:
 
It is the central part of the answer and one should understand the demand of the question to provide rich content.

AI's Impact on Healthcare:

AI, powered by machine learning and big data analytics, is reshaping the healthcare landscape by reducing costs and improving outcomes. It enhances clinical diagnosis, disease detection, and treatment efficacy, transforming traditional healthcare practices into more patient-centric approaches.

Examples:

Transplant Surgery Without Blood Transfusion: Before transplant surgery, AI evaluates blood coagulation factors to ensure optimal conditions for surgery. By monitoring blood thinners and coagulation effects in real-time during surgery, AI technology facilitates safe surgeries without the need for blood transfusions.
 
Early Cancer Detection: AI plays a crucial role in early cancer detection by analyzing radiological and pathological images to identify cancerous traits. Through machine learning algorithms trained on extensive datasets, AI detects tissue changes indicative of malignancies, enabling early intervention and improved patient outcomes.
 
Tumour Picture Bank at TMH: TMH utilizes AI to create a tumour picture bank, annotating and categorizing images based on biopsy results, histology, and genetic sequences. These algorithms aid in treatment response assessment, guiding personalized treatment plans and minimizing unnecessary chemotherapy.
 
Conclusion: 
 
 The ending of the answer should be on a positive note and it should have a forward-looking approach.
 
Artificial intelligence is revolutionizing the healthcare sector, enhancing diagnosis, treatment, and patient care. Through innovative applications like TMH's 'Bio-Imaging Bank,' AI empowers healthcare professionals to make more informed decisions, leading to better outcomes and improved patient experiences. As AI continues to evolve, its potential to transform healthcare delivery and improve population health outcomes remains promising.
 
 
Other Points to Ponder 

Can AI help reduce cancer fatalities in the future?

Bio-Imaging Bank

 

Previous Year Questions

1. Introduce the concept of Artificial Intelligence (AI). How does AI help clinical diagnosis? Do you perceive any threat to privacy of the individual in the use of AI in healthcare? (2023)

2. What do you understand by nanotechnology and how is it helping in health sector? (2020)

 

(mains - general-studies-3 ) 05-Apr 2024
Question:

The PSLV Orbital Experimental Module-3 has successfully achieved all its objectives. Discuss.

 

Introduction:

A simple introduction about PSLV Orbital Experimental Module-3 

The PSLV Orbital Experimental Module-3 (POEM-3), an innovative low-cost space platform based on the spent PS4 stage of the PSLV-C58 vehicle, has successfully achieved all its objectives.

After deploying the XPoSat satellite into its intended 650 km orbit on January 1, 2024, the vehicle was manoeuvred to a lower 350 km circular orbit to minimize orbit degradation once the experiment concluded.

POEM-3 is a three-axis attitude-controlled platform equipped with power generation, telecommand, and telemetry capabilities to support various payloads.

 

Body:

You may incorporate some of the following points in the body of your answer:

POEM-3 carried nine payloads from diverse entities such as the VSSC, PRL, academia, and space start-ups inducted through IN-SPACe. These payloads include:

  • The Women Engineered Satellite (WESAT) was developed by the LBS Institute of Technology for Women to study the comparison between solar irradiance and UV index.
  • A radiation shielding experiment.
  • An amateur radio.
  • Three propulsion systems by space start-ups, and an interplanetary dust count experiment by the Physical Research Laboratory.
  • Two types of fuel cell systems developed by the Vikram Sarabhai Space Centre. One of the fuel cells utilizes hydrogen and oxygen to generate electricity while releasing heat and water as by-products, serving as a precursor for energy systems required for a space station.

 

Conclusion:

Your Conclusion should be simple and balanced

Having accomplished all its objectives, further tests with POEM-3 are planned to generate data for future missions, including forthcoming POEM configurations. With POEM-3's expected orbital decay and return in three months, the PSLV-C58 XPoSat mission will leave no debris in space.

India's aspirations of setting up a space station in low Earth orbit by 2035 align with the successful demonstration of POEM-3, showcasing the country's growing capabilities in space technology and exploration.

 

Other Points to Ponder

ARKA200 (Xenon Based Electric Propulsion)
RUDRA (HAN-based Green Propellant Thruster)
Other space missions of ISRO

 

Previous Year Questions

1. What is the main task of India’s third moon mission which could not be achieved in its earlier mission? List the countries that have achieved this task. Introduce the subsystems in the spacecraft launched and explain the role of the Virtual Launch Control Centre’ at the Vikram Sarabhai Space Centre which contributed to the successful launch from Sriharikota. (2023)

2. Launched on 25th December 2021, James Webb Space Telescope has been much in the news since then. What are its unique features which make it superior to its predecessor Space Telescopes? What are the key goals of this mission? What potential benefits does it hold for the human race? (2022)

 

(mains - general-studies-3 ) 05-Apr 2024
Question:

What are melanistic tigers and what makes tigers melanistic? How setting up a melanistic tiger safari near Similipal Tiger Reserve (STR) will boost tourism?

 

Introduction:

 A Simple Introduction about Melanistic Tigers

Melanism is a genetic condition characterized by increased melanin production, resulting in black or nearly black skin, feathers, or hair in animals. In the Similipal Tiger Reserve (STR), there exists a unique lineage of royal Bengal tigers with higher-than-normal levels of melanin, giving them black and yellow interspersed stripes on their coats, though they are more accurately described as pseudo-melanistic.

Body:

You may incorporate some of the following points in the body of your answer:

What makes tigers melanistic?

  • Researchers at the National Centre for Biological Sciences (NCBS) in Bengaluru have identified a single mutation in the gene Transmembrane Aminopeptidase Q (Taqpep) responsible for the unique appearance of melanistic tigers in Similipal. This mutation causes the black tigers to develop stripes that appear broader or spread into the tawny background.
  • Genetic analysis and computer simulations suggest that the pseudo-melanistic tigers in Similipal are inbred and descended from a small founding population of tigers. These tigers live in isolation within the STR, leading to inbreeding among themselves.

Setting up a melanistic tiger safari near Similipal Tiger Reserve (STR):

  • The STR is the only place where melanistic tigers have been recorded in Odisha. According to the 2022 All India Tiger Estimation, 16 individuals were recorded in STR, with 10 being melanistic.
  • Susanta Nanda, Principal Chief Conservator of Forests (Wildlife), Odisha, has identified approximately 200 hectares of land along the Dhanbad-Balasore National Highway-18 for the proposed safari. This area is about 15 kilometres from STR and offers a scenery similar to Simlipal.
  • The safari aims to allow wildlife conservationists, researchers, and enthusiasts to observe the rare melanistic tigers up close, raising awareness about the need for their conservation.
  • Since tiger sightings in STR can be challenging due to its vast area, the safari is proposed as an additional attraction for visitors to Similipal.

Conclusion:

Your conclusion should be simple and balanced

In addition to the identification of suitable land for the safari, the state government will need to obtain various statutory clearances, including approval from the Central Zoo Authority. The proposed melanistic tiger safari near Similipal Tiger Reserve holds promise for boosting tourism while contributing to the conservation efforts of these unique big cats.

 

Other Points to Ponder

Tiger Reserves in India

NTCA

 

Previous Year Questions

1. Comment on the National Wetland Conservation Programme initiated by the Government of India and name a few India’s wetlands of international importance included in the Ramsar Sites. (2023)

2. How does biodiversity vary in India? How is the Biological Diversity Act, 2002 helpful in conservation of flora and fauna? (2018)

 

(mains - general-studies-3 ) 05-Apr 2024
Question:
What is geospatial intelligence? How will geospatial solutions address national priorities and SDGs?
 

Introduction:

The introduction of the answer is essential and should be restricted to 3-5 lines. Remember, a one-liner is not a standard introduction

Geospatial intelligence involves gathering and amalgamating data from diverse technological sources such as satellites, mobile sensors, ground control stations, and aerial photography. This data is utilized to create real-time maps and simulations, aiding in forecasting threats in terms of their location, timing, and magnitude. Stakeholders ranging from individuals to government officials utilize this information to make informed decisions.

Body:

 It is the central part of the answer and one should understand the demand of the question to provide rich content

Various initiatives, including the Sustainable Development Goals (SDGs) and National Developmental Priorities, address the integration of geospatial data into research and operational endeavors. The National Programme on Spatial Disaster Risk Reduction (SDRR) exemplifies this approach, encompassing initiatives such as Landslide Hazard Mitigation (LHM) and Coastal Hazard, Risk Assessment and Reduction (CHRAR). LHM focuses on developing early warning systems for landslides in vulnerable regions like Sikkim, Nilgiris, Manikaran, and Uttarakhand. CHRAR, on the other hand, employs geospatial techniques to study coastal disasters, including erosion, tsunamis, cyclones, and floods, aiming to produce actionable outputs for relief efforts.

In urban governance, geospatial solutions play a crucial role, with emphasis on emerging technologies like Geo-ICT. Projects under this domain focus on areas such as IoT and GIS technologies, smart parking management systems, flood mapping frameworks, and indoor navigation systems, contributing to the development of smart cities.

Conclusion:

The ending of the answer should be on a positive note and it should have a forward-looking approach

The recent signing of the Indo-US Basic Exchange and Cooperation Agreement (BECA) underscores the importance of geospatial intelligence in national security. This agreement enables India to leverage US geospatial intelligence, enhancing the accuracy of automated systems and weapons like missiles and armed drones. Access to topographical and aeronautical data through BECA will significantly bolster India's capabilities in navigation and targeting, marking a significant advancement in defense collaboration between the two nations

 

Other Points to Consider

Geospatial Technology Programme

Geospatial Science Programme

 

 

Previous Year Questions

What do you understand by nanotechnology and how is it helping in health sector? (2020)

With growing energy needs should India keep on expanding its nuclear energy programme? Discuss the facts and fears associated with nuclear energy? (2018)

(mains - general-studies-3 ) 04-Apr 2024
Question:
What is Micro Irrigation, and what are its benefits? Discuss the government’s efforts related to micro irrigation.
 

Introduction:

The introduction of the answer is essential and should be restricted to 3-5 lines. Remember, a one-liner is not a standard introduction

Micro Irrigation represents a holistic approach aimed at boosting crop productivity by enhancing water utilization efficiency through Micro Irrigation Systems.

It involves the gradual delivery of small drips, minute streams, or miniature sprays of water either above or below the soil surface.

Micro irrigation systems are adept at conserving water and improving water utilization efficiency compared to conventional surface irrigation methods.

Benefits

(i) Decreases water consumption

(ii) Suppresses the growth of undesirable vegetation (weeds)

(iii) Applicable across diverse types of terrain

(iv) Suited for undulating landscapes

(v) Enhances crop yield and reduces labor expenses

(vi) Diminishes the need for fertilizers while enhancing soil fertility.

Body:

It is the central part of the answer and one should understand the demand of the question to provide rich content

Government Initiatives

Per Drop More Crop (PDMC) scheme

This initiative, spearheaded by the Department of Agriculture & Farmers’ Welfare, is a central government endeavor promoting micro-irrigation systems like drip or sprinkler systems.

Initiated in 2006, it was subsequently absorbed into the Pradhan Mantri Krishi Sichai Yojna (PMKSY) in 2015, forming one of its integral components.

Financing is divided, with the central government contributing approximately 40 percent, and the remainder financed by states and farmers themselves, with varying proportions in hilly and northeastern states.

The scheme primarily aims to augment water-use efficiency and bolster productivity, thereby augmenting farmer incomes.

Pradhan Mantri Krishi Sinchayee Yojana (PMKSY)

This scheme, undertaken by the Government of India, endeavors to enhance water utilization efficiency in agriculture by advocating suitable technological interventions.

Micro Irrigation Fund

This initiative is committed to facilitating agricultural irrigation access by prioritizing water conservation and management.

It offers several advantages such as increased subsidies on micro irrigation systems, complimentary service for three years post-installation of micro irrigation systems, and heightened water utilization efficiency.

States can access the fund in the form of a loan from NABARD (National Bank for Agriculture and Rural Development) at a 3 percent lower cost of funds, with the central government compensating the remaining 3 percent

 

Conclusion:

The ending of the answer should be on a positive note and it should have a forward-looking approach

Micro Irrigation, bolstered by government initiatives like the Per Drop More Crop (PDMC) scheme and the Pradhan Mantri Krishi Sinchayee Yojana (PMKSY), emerges as a pivotal tool in enhancing agricultural productivity while conserving water resources. Through the effective implementation of micro irrigation systems, farmers can achieve higher crop yields, reduce water consumption, and mitigate the growth of unwanted vegetation. Moreover, initiatives such as the Micro Irrigation Fund further incentivize the adoption of these systems by providing financial assistance and promoting water conservation practices. By leveraging micro irrigation technologies and government support, India can progress towards sustainable agriculture, ensuring food security, and bolstering the livelihoods of farmers nationwide

 

Other Points to Consider

Sprinkler and Drip Irrigation

Challenges in implementation

 

 

Previous Year Questions

How and to what extent would micro-irrigation help in solving India’s water crisis? (2021)

Suggest measures to improve water storage and irrigation system to make its judicious use under depleting scenario. (2020)

What is Integrated Farming System? How is it helpful to small and marginal farmers in India? (2022)

(mains - general-studies-3 ) 04-Apr 2024
Question:
Describe the blue economy and explain its significance. Does India have a blue economy policy?
 

Introduction:

The introduction of the answer is essential and should be restricted to 3-5 lines. Remember, a one-liner is not a standard introduction

The concept of the "blue economy" broadly encompasses economic activities linked to oceans and coastlines, often with an implicit emphasis on sustainability.

According to the European Commission, it encompasses "all economic activities related to oceans, seas, and coasts, encompassing a wide array of interconnected established and emerging sectors."

The World Bank defines the blue economy as the "sustainable utilization of ocean resources to foster economic growth, enhance livelihoods, and create employment opportunities while safeguarding the health of ocean ecosystems."

Body:

It is the central part of the answer and one should understand the demand of the question to provide rich content

For countries like India, endowed with extensive coastlines, abundant marine resources, and diverse tourism potential, the blue economy holds significant importance.

Finance Minister Sitharaman highlighted the launch of a scheme focusing on restoration, adaptation measures, coastal aquaculture, and mariculture, adopting an integrated, multi-sectoral approach.

Restoration and adaptation efforts are vital to ensuring that economic activities do not compromise the health of ocean ecosystems.

Does India have a blue economy policy?

A preliminary draft policy framework for India's Blue Economy was unveiled in July 2022.

As per the Press Information Bureau (PIB), the policy blueprint outlined key recommendations covering aspects such as the National Accounting Framework for the Blue Economy, Ocean Governance, Coastal Marine Spatial Planning, Tourism Prioritization, Marine Fisheries, Aquaculture, Fish Processing, Manufacturing, Emerging Industries, Trade, Technology, Services, Skill Development, Logistics, Infrastructure, Shipping, Coastal and Deep-Sea Mining, Offshore Energy, Security, Strategic Dimensions, and International Engagement.

Conclusion:

The ending of the answer should be on a positive note and it should have a forward-looking approach

The Finance Minister additionally announced plans to establish five integrated aquaparks and expand the Pradhan Mantri Matsya Sampada Yojana (PMMSY) to increase aquaculture productivity, double exports to Rs 1 lakh crore, and create 55 lakh employment opportunities in the near future, as reported by ANI

Other Points to Consider

Coastal aquaculture

Mariculture

 

 

Previous Year Questions

Explain the causes and effects of coastal erosion in India. What are the available coastal management techniques for combating the hazard? (2022)

Explain various types of revolutions, took place in Agriculture after Independence in India. How these revolutions have helped in poverty alleviation and food security in India? (2017)

(mains - general-studies-3 ) 04-Apr 2024
Question:
What is nano di-ammonium phosphate (Nano DAP)? Why does the government want to expand its use?
 

Introduction:

The introduction of the answer is essential and should be restricted to 3-5 lines. Remember, a one-liner is not a standard introduction

DAP, known as di-ammonium phosphate, ranks as the second most utilized fertilizer in India following urea.

It boasts a high phosphorus content that plays a vital role in stimulating root establishment and development. Without adequate phosphorus, plants struggle to attain their usual size or experience delayed maturation.

Nano DAP, comprising 8% Nitrogen and 16% Phosphorus by volume, diverges from conventional DAP, which typically exists in granular form. Notably, the Nano DAP manufactured by the Indian Farmers Fertiliser Cooperative (IFFCO) is presented in liquid form.

Body:

It is the central part of the answer and one should understand the demand of the question to provide rich content

Rationale for Government Interest:

The minute particle size of Nano DAP renders it more efficient compared to its traditional counterpart. This efficiency allows the fertilizer to penetrate easily into the seed surface or through plant openings such as stomata.

Enhanced assimilation of the fertilizer within the plant system translates to augmented seed vigor, heightened chlorophyll levels, enhanced photosynthetic efficiency, improved crop quality, and amplified yields.

Given the substantial subsidies offered by the government on DAP, transitioning to a more economical fertilizer variant is poised to alleviate the government's subsidy burden significantly.

Advantages:

Nano DAP proves to be more economically viable.

Its liquid form renders Nano DAP considerably more convenient for application.

Conclusion:

The ending of the answer should be on a positive note and it should have a forward-looking approach

Most notably, India presently relies heavily on fertilizer imports to meet domestic demand. The widespread adoption of domestically produced Nano DAP, manufactured in Kalol, Gujarat, is anticipated to markedly alleviate this dependence on imports.

This groundbreaking initiative not only propels Indian agriculture forward in food grain production but also bolsters India's self-reliance in fertilizer manufacturing.

The integration of Nano DAP into agricultural practices stands to foster self-sufficiency in fertilizers, thereby offering substantial benefits to our farming community

 

Other Points to Consider

IFFCO

Nano DAP

 

 

Previous Year Question

How do subsidies affect the cropping pattern, crop diversity and economy of farmers? What is the significance of crop insurance, minimum support price and food processing for small and marginal farmers? (2017)

(mains - general-studies-3 ) 04-Apr 2024
Question:
What are high-altitude pseudo-satellite vehicles (HAPS)? Discuss the challenges of HAPS
 

Introduction:

The introduction of the answer is essential and should be restricted to 3-5 lines. Remember, a one-liner is not a standard introduction

The concept of High Altitude Pseudo Satellite (HAPS) refers to an advanced solar-powered unmanned aerial system designed to sustain prolonged control over a designated area, positioned above the troposphere, and equipped with specialized mission sensors. Operating at altitudes around 20 kilometers, solely powered by solar energy, and capable of enduring in-flight for several months, HAPS presents a revolutionary tool with multifaceted applications, notably in surveillance, monitoring, and disaster management scenarios.

Body:

It is the central part of the answer and one should understand the demand of the question to provide rich content

The National Aerospace Laboratories (NAL) in Bengaluru recently achieved a milestone by conducting the inaugural test flight of a solar-powered "pseudo satellite," a cutting-edge unmanned aerial vehicle (UAV) poised to elevate India's surveillance capabilities, particularly in border regions. Despite being a nascent technology, India's successful test flight positions it alongside a select group of nations exploring HAPS technology.

Challenges of HAPS:

Developing a self-sustaining solar-powered aircraft capable of extended flight durations presents formidable technological challenges. Key hurdles include generating sufficient solar energy to power the aircraft, sustain payload operations, and recharge batteries throughout the night. Additionally, design considerations must address the aircraft's lightweight construction to minimize power demands while ensuring stability. Operating in frigid temperatures exceeding -50 degrees Celsius at high altitudes necessitates specialized measures to maintain electronic functionality.

Conclusion:

The ending of the answer should be on a positive note and it should have a forward-looking approach

Unlike conventional battery-powered UAVs limited by short flight durations and restricted coverage, HAPS represents a transformative advancement akin to geostationary satellites but with added versatility. Its capacity for redeployment to different locations and adaptability to diverse payloads distinguishes it from geostationary satellites, offering unprecedented flexibility in aerial surveillance and monitoring applications

Other Points to Consider

What is the need for such UAVs?

India and the HAPS

 

 

Previous Year Question

What is the main task of India’s third moon mission which could not be achieved in its earlier mission? List the countries that have achieved this task. Introduce the subsystems in the spacecraft launched and explain the role of the Virtual Launch Control Centre’ at the Vikram Sarabhai Space Centre which contributed to the successful launch from Sriharikota. (2023)

(mains - general-studies-3 ) 04-Apr 2024
Question:
What is the Square Kilometer Array project? What benefits will joining it bring to the scientific community in India?
 

Introduction:

The introduction of the answer is essential and should be restricted to 3-5 lines. Remember, a one-liner is not a standard introduction

The Square Kilometre Array (SKA) project represents a monumental international collaboration aimed at constructing the world's most extensive radio telescope, boasting a cumulative collecting area exceeding one million square meters. Unlike traditional single large telescopes, the SKA comprises thousands of dish antennas meticulously orchestrated to function as a unified entity. Situated in sparsely populated areas across South Africa and Australia, the antennas are strategically dispersed to minimize interference from human activities. Once operational, the SKA is poised to significantly surpass existing radio telescopes in potency, promising groundbreaking insights into the cosmos

Body:

You may incorporate some of the following points in the body of your answer:

The Square Kilometre Array (SKA) project represents a collaborative endeavor on a global scale aimed at constructing the most extensive radio telescope worldwide, boasting a cumulative collecting area exceeding one million square meters.

Unlike traditional single large telescopes, the SKA comprises thousands of dish antennas meticulously orchestrated to function as a unified entity.

Strategically located in sparsely populated areas, the antennas are dispersed across South Africa and Australia, with approximately 200 and over 130,000 installations, respectively, chosen to minimize interference from human activities.

Once operational, the SKA is poised to outperform existing radio telescopes by a considerable margin, exhibiting a potency ranging from 5 to 60 times that of current counterparts operating within similar frequency spectrums.

Benefits for India

India has officially committed to participating in the Square Kilometer Array (SKA) initiative, a pivotal global scientific collaboration endeavoring to construct the world's largest radio telescope.

Although none of the SKA infrastructure will be situated within Indian territory, the country stands to reap substantial scientific and technological dividends by assuming full membership in the project.

While India has been actively involved in the SKA project for several years, attaining full member status entails formal ratification of an international treaty and a corresponding financial commitment, thereby unlocking enhanced scientific prospects with the impending facility.

Comparable to the scientific opportunities presented by initiatives such as the LHC or ITER, despite being situated abroad, the SKA membership offers India preferential access to the facilities and resources of the project.

As a full member, India stands to gain preferential allocation of time on the radio telescope, commensurate with its contribution to the endeavor, with limited additional time slots accessible via competitive selection processes.

Conclusion:

The ending of the answer should be on a positive note and it should have a forward-looking approach

India has allocated Rs 1,250 crore towards the SKA project, earmarking its financial commitment for the construction phase of the initiative.

In addition to its involvement in the SKA, India has embarked on constructing a gravitational wave detector to augment the international LIGO network and holds full membership status in the ITER project, focusing on harnessing energy from nuclear fusion reactions.

Furthermore, India's robust engagement in initiatives such as the Large Hadron Collider (LHC), the world's largest and most potent particle accelerator, underscores its proactive participation in groundbreaking scientific endeavors at the global forefront

Other Points to Consider

India’s involvement

Laser Interferometer Gravitational Wave Observatory

ITER project

 

 

Previous Year Questions

Launched on 25th December, 2021, James Webb Space Telescope has been much in the news since then. What are its unique features which make it superior to its predecessor Space Telescopes? What are the key goals of this mission? What potential benefits does it hold for the human race? (2022)

Discuss the work of ‘Bose-Einstein Statistics’ done by Prof. Satyendra Nath Bose and show how it revolutionized the field of Physics. (2018)

(mains - general-studies-3 ) 04-Apr 2024
Question:
What is a Large Language Model, or LLM? How many types of LLMs are there and how do they work?
 

Introduction:

 The introduction of the answer is essential and should be restricted to 3-5 lines. Remember, a one-liner is not a standard introduction

Large Language Models (LLMs), as defined by Google, are expansive general-purpose models capable of being pre-trained and subsequently fine-tuned for specific tasks.

These models are equipped to tackle common language challenges including text classification, question answering, cross-industry text production, document summarization, and more.

LLMs can also be customized to address specific issues across various domains such as finance, retail, entertainment, etc., often utilizing relatively small sets of field-specific datasets.

Body:

You may incorporate some of the following points in the body of your answer:

Types of LLMs

LLMs can be classified in several ways:

(i) Architecturally, they fall into three categories: autoregressive, transformer-based, and encoder-decoder models.

(ii) In terms of training data, there are three types: pretrained and fine-tuned, multilingual models capable of understanding and generating text in multiple languages, and domain-specific models trained on data relevant to particular fields such as legal, finance, or healthcare.

(iii) They can also be categorized as open-source or closed-source based on availability; while some are publicly accessible, others remain proprietary. Examples of open-source LLMs include LLaMA2, BlOOM, Google BERT, Falcon 180B, and OPT-175 B, whereas proprietary LLMs include Claude 2, Bard, and GPT-4.

How LLMs Operate?

LLMs operate on the principle of "deep learning", involving the construction of artificial neural networks inspired by the structure and functioning of the human brain.

These neural networks are trained to predict the probability of a word or sequence of words given the preceding words in a sentence for LLMs.

Conclusion:

The ending of the answer should be on a positive note and it should have a forward-looking approach

LLMs excel in predicting the most probable next word or sequence of words based on input prompts.

With their versatility, LLMs find applications across diverse domains, generating text for various purposes such as articles, songs, poems, and stories, while also serving as virtual assistants or engaging in conversations

 

Other Points to Consider

ChatGPT

Gemini

What are the advantages of LLMs?

 

Previous Year Questions

Introduce the concept of Artificial Intelligence (AI). How does AI help clinical diagnosis? Do you perceive any threat to privacy of the individual in the use of AI in healthcare? (2023)

What do you understand by nanotechnology and how is it helping in health sector? (2020)

(mains - general-studies-3 ) 04-Apr 2024
Question:
What is Fair and Remunerative Price for Sugarcane? Discuss how FRP differs from MSP
 

Introduction:

 The introduction of the answer is essential and should be restricted to 3-5 lines. Remember, a one-liner is not a standard introduction

The Fair and Remunerative Price (FRP) for sugarcane is determined by the Cabinet Committee on Economic Affairs (CCEA) at the central level.

This price is calculated based on the sugar recovery rate from the cane, which represents the percentage of sugar extracted relative to the amount of cane crushed. A higher recovery rate translates to a higher FRP, reflecting increased sugar production from the cane.

Body:

You may incorporate some of the following points in the body of your answer:

Sugar mills are legally obligated to pay the FRP to sugarcane farmers for their produce. Failure to settle FRP dues within 14 days of cane sale may lead to enforcement action by cane commissioners. Non-payment could result in the seizure of mill assets as outstanding land tax liabilities.

How does FRP differ from the Minimum Support Price (MSP)?

MSPs serve as a safety net for farmers, ensuring a minimum guaranteed payment to mitigate risks associated with agriculture, such as adverse weather conditions.

They provide stability in market prices, preventing fluctuations that could impact both farmers' incomes and consumers' access to essential food items.

The government announces MSPs annually for specific crops, indicating the price at which it will procure the produce from farmers if market prices fall below the set MSP.

MSPs are determined based on various factors including production costs, demand-supply dynamics, market trends, and ensuring a minimum profit margin for farmers, typically set at 50% over production costs.

Conclusion:

The ending of the answer should be on a positive note and it should have a forward-looking approach

The central government introduced MSP for sugar in 2018 as part of measures to stabilize sugar prices and maintain a balanced demand-supply ratio.

Additionally, the government has established mill-specific sales quotas. Non-compliance with these regulations could lead to punitive measures under the Essential Commodities Act of 1955, including fines, imprisonment, or both, as a means to regulate the sugar industry effectively

 

Other Points to Consider

Why does sugarcane have both the FRP and MSP?

 

 

Previous Year Questions

What do you mean by Minimum Support Price (MSP)? How will MSP rescue the farmers from the low income trap? (2018)

What are the challenges and opportunities of the food processing sector in the country? How can the income of the farmers be substantially increased by encouraging food processing? (2020)

(mains - general-studies-3 ) 04-Apr 2024
Question:
What is WTO Agreement on Agriculture? Discuss the broad areas of agriculture and trade policy
 

Introduction:

The introduction of the answer is essential and should be restricted to 3-5 lines. Remember, a one-liner is not a standard introduction

The Uruguay Round global trade negotiations, formally endorsed in Marrakesh, Morocco, in April 1994, ushered in various agreements, including the WTO Agreement on Agriculture, which commenced implementation from January 1, 1995. This agreement delineated specific reduction obligations for different categories of countries, emphasizing market access, domestic support, and export subsidies.

Body:

It is the central part of the answer and one should understand the demand of the question to provide rich content

Market Access:

Provisions concerning market access encompassed tariffication, tariff reductions, and access opportunities. Industrialized nations were mandated to reduce their ordinary tariffs by an average of 36% over a six-year period, with each tariff item subjected to a minimum reduction of 15%. Conversely, developing nations, still grappling with quantitative restrictions due to balance-of-payments concerns, were permitted to offer ceiling bindings instead of tariffs.

Domestic Support:

Regarding domestic support measures, which were subject to reduction commitments, affluent nations were required to curtail overall support provided during 1986-88 by 20%, while developing countries faced a reduction of 13.3%. Certain policies falling under domestic support categories, constituting less than 5% of the value of production for developed countries and less than 10% for developing nations, were exempt from reduction commitments.

Export Subsidies:

The Agreement mandated reductions in export subsidies, with developed countries tasked with slashing export subsidy expenditure by 36% and volume by 21% over six years, based on 1986-1990 levels. Similarly, developing countries were required to make cuts of 24% and 14%, respectively, over a decade. Furthermore, the Agreement prohibited future subsidies for products not subject to export subsidy reduction agreements.

Conclusion:

The ending of the answer should be on a positive note and it should have a forward-looking approach

Although the WTO’s ministerial conference concluded without resolving crucial issues pertinent to India's interests, such as devising a permanent solution for the public food Public Stockholding (PSH) program and curbing fisheries subsidies contributing to overcapacity and overfishing, members agreed to extend the moratorium on imposing import duties on e-commerce trade for an additional two years. These deliberations underscore the ongoing challenges and negotiations within the WTO framework

 

Other Points to Consider

13th ministerial conference (MC)

India’s commitment

India’s PSH (Public Stockholding) programme

 

 

Previous Year Questions

What are the direct and indirect subsidies provided to farm sector in India? Discuss the issues raised by the World Trade Organization (WTO) in relation to agricultural subsidies. (2023)

How would the recent phenomena of protectionism and currency manipulations in world trade affect macroeconomic stability of India? (2018)

(mains - general-studies-3 ) 04-Apr 2024
Question:
There is a significant increase in India’s leopard population as per Status of Leopards in India 2022. Discuss the methodology and outcomes of the report
 

Introduction:

The introduction of the answer is essential and should be restricted to 3-5 lines. Remember, a one-liner is not a standard introduction

The Indian leopard (Panthera pardus fusca) is found across diverse forested habitats spanning India, Nepal, Bhutan, and certain regions of Pakistan. Classified as "Vulnerable" by the IUCN Red List and listed in Schedule I of the Wildlife (Protection) Act, 1972, this species faces conservation challenges.

According to recent findings, India's leopard population increased from 12,852 in 2018 to 13,874 in 2022.

Body:

It is the central part of the answer and one should understand the demand of the question to provide rich content

Methodology:

The fifth cycle of leopard population assessment in India (2022) focused on forested habitats across 18 tiger states, encompassing four significant tiger conservation landscapes. Utilizing a likelihood-based spatially explicit capture mark-recapture (SECR) covariate model, researchers combined photo-captures with geographical data on prey, habitat, and human-induced factors to estimate leopard abundance.

Phase 1:

Systematic sampling of wooded regions within each landscape was conducted using M-STrIPES Android apps and desktop applications.

Phase 2:

Remote sensing and secondary data were employed to model leopard occupancy and abundance in response to habitat factors and human influences.

Phase 3:

SECR models were utilized to predict leopard density through camera trap data, with individuals identified using image and pattern processing applications such as CaTRACT and ExtractCompare.

Outcomes:

Madhya Pradesh boasts the highest leopard count in the country, with 3,907 individuals (up from 3,421 in 2018). Central India harbors the most significant population, comprising 8,820 leopards, followed by the Western Ghats with 3,596, and the Shivalik Hills and Gangetic Plains with 1,109 individuals. Population growth rates varied across regions, with a decline observed in the Shivalik Hills and Gangetic Plains, while other regions experienced moderate increases.

In Central India, a substantial proportion (68%) of leopards reside in human-use forests outside Protected Areas, heightening vulnerability to poaching and human-leopard conflicts. Similarly, in the Western Ghats, although leopard populations remain stable, a significant portion (65%) inhabits areas outside Protected Areas.

Conclusion:

The ending of the answer should be on a positive note and it should have a forward-looking approach

Conservation efforts for the Indian leopard necessitate a multifaceted approach, including habitat restoration initiatives, protection measures beyond Protected Areas, and the implementation of strategies to mitigate human-leopard conflicts. Records maintained by the Wildlife Protection Society of India (WPSI) underscore the urgency of addressing poaching threats, with over 5,500 leopards poached since 1994. Efforts aimed at safeguarding this species are imperative to ensure its long-term survival and ecological balance

Other Points to Ponder

Benefits from tiger conservation efforts

Leopard-human conflict remains a worry

 

 

Previous Year Questions

How does the draft Environment Impact Assessment (EIA) Notification, 2020 differ from the existing EIA Notification, 2006? (2020)

The Intergovernmental Panel on Climate Change (IPCC) has predicted a global sea level rise of about one metre by AD 2100. What would be its impact in India and the other countries in the Indian Ocean region? (2023)

(mains - general-studies-3 ) 04-Apr 2024
Question:
What is MethaneSAT? What are the features of MethaneSAT? Discuss the need to track and measure methane emissions.
 

Introduction:

The introduction of the answer is essential and should be restricted to 3-5 lines. Remember, a one-liner is not a standard introduction

MethaneSAT, a satellite engineered to track and quantify methane emissions on a global scale, was successfully launched aboard a SpaceX Falcon9 rocket from California. Orbiting the Earth 15 times daily, MethaneSAT will primarily focus on monitoring emissions from the oil and gas sector. It aims to gather extensive data to discern the sources of methane, ascertain responsibility for emissions, and assess whether these emissions are escalating or diminishing over time. The data collected by MethaneSAT will be publicly accessible in near real-time, facilitating prompt action by stakeholders and regulators to mitigate methane emissions.

Body:

It is the central part of the answer and one should understand the demand of the question to provide rich content

Key Features of MethaneSAT:

MethaneSAT is outfitted with a sophisticated high-resolution infrared sensor and a spectrometer, enabling it to fulfill crucial data requirements. It possesses the capability to detect minute variations in methane concentrations, reaching as low as three parts per billion in the atmosphere, thereby enabling the identification of smaller emission sources compared to preceding satellites. Additionally, MethaneSAT boasts a wide-camera view, spanning approximately 200 km by 200 km, facilitating the identification of larger emitters, colloquially referred to as "super emitters."

Need to Track and Quantify Methane Emissions:

Methane, despite being invisible, ranks as a potent greenhouse gas and stands as the second-largest contributor to global warming after carbon dioxide, responsible for 30% of global heating since the onset of the Industrial Revolution. With methane being 80 times more potent at trapping heat in the atmosphere than carbon dioxide over a 20-year period, its emissions pose significant environmental challenges. Furthermore, methane contributes to the formation of ground-level ozone, a hazardous gas linked to approximately one million premature deaths annually. Notably, the burning of fossil fuels constitutes a primary source, accounting for roughly 40% of human-induced methane emissions.

Conclusion:

The ending of the answer should be on a positive note and it should have a forward-looking approach. 

In 2021, over 150 nations committed to the Global Methane Pledge, pledging to reduce their collective methane emissions by at least 30% from 2020 levels by 2030. Moreover, more than fifty businesses pledged to substantially curtail routine flaring and methane emissions. MethaneSAT is poised to play a pivotal role in supporting these commitments by providing crucial data to aid in tracking progress and implementing measures to mitigate methane emissions effectively

 

Other Points to Consider

Why is it significant?

Ozone

Environmental Defense Fund (EDF)

 

Previous Year Question

What is the main task of India’s third moon mission which could not be achieved in its earlier mission? List the countries that have achieved this task. Introduce the subsystems in the spacecraft launched and explain the role of the Virtual Launch Control Centre’ at the Vikram Sarabhai Space Centre which contributed to the successful launch from Sriharikota. (2023)

(mains - general-studies-3 ) 04-Apr 2024
Question:
Discuss about India’s indigenous fifth-gen fighter jet Advanced Medium Combat Aircraft (AMCA). Why it is important?
 

Introduction:

The introduction of the answer is essential and should be restricted to 3-5 lines. Remember, a one-liner is not a standard introduction

The Aeronautical Development Agency (ADA) under the auspices of the Defence Research and Development Organisation (DRDO) has been designated as the lead agency for overseeing the program and conceptualizing the aircraft. Hindustan Aeronautics Limited (HAL), a state-owned enterprise, will undertake the manufacturing process. This initiative will position India among the select few nations possessing their own fifth-generation fighter aircraft.

Body:

It is the central part of the answer and one should understand the demand of the question to provide rich content

Features of AMCA:

The forthcoming 25-tonne twin-engine aircraft, larger than existing fighters in the Indian Air Force's arsenal, will incorporate advanced stealth capabilities to evade detection by enemy radar systems. Noteworthy features include a concealed internal fuel tank with a capacity of 6.5 tonnes and an internal weapons bay capable of housing an array of weapons, including indigenous armaments, discreetly within its fuselage. The initial variant, AMCA Mk1, will be powered by the US-built GE414 engine with a thrust rating of 90 kilonewtons (kN), while the more advanced AMCA Mk2 will utilize a domestically developed 110kN engine, a collaborative effort between DRDO's Gas Turbine Research Establishment (GTRE) and a foreign defense major. Additionally, the aircraft will incorporate diverterless supersonic inlets for efficient control of airflow into the engines.

Significance:

Originally conceived as a joint development venture with Russia under the Fifth Generation Fighter Aircraft (FGFA) program, India withdrew from the collaboration in 2018, paving the way for the AMCA to emerge as India's indigenous fifth-generation fighter aircraft. While the indigenous Light Combat Aircraft (LCA) Tejas represents a 4.5-generation single-engine multirole aircraft, the AMCA will mark a significant leap forward. With its low electromagnetic signature, the AMCA will pose challenges for enemy radar detection, thanks to its stealth features including internal weapons storage and a sizable internal fuel tank. The use of specialized materials on the aircraft's surface will divert radar signatures, enhancing its stealth capabilities and ensuring longer operational durations with reduced maintenance requirements.

Conclusion:

The ending of the answer should be on a positive note and it should have a forward-looking approach

The Cabinet Committee on Security (CCS) has approved a project worth Rs 15,000 crore for the development of the Advanced Medium Combat Aircraft (AMCA), a noteworthy endeavor aimed at establishing India's prowess in the domain of fifth-generation multirole fighter jets. Dr. Krishna Rajendra Neeli, project director of AMCA at ADA, has expressed confidence that the aircraft will rival or surpass other fifth-generation stealth fighter aircraft currently in service worldwide

Other Points to Consider

Cabinet Committee on Security

Aeronautical Development Agency

Advanced Medium Combat Aircraft (AMCA)

 

Previous Year Questions

 

How is S-400 air defence system technically superior to any other system presently available in the world? (2021)

Discuss India’s achievements in the field of Space Science and Technology. How the application of this technology has helped India in its socio-economic development? (2016)

(mains - general-studies-3 ) 04-Apr 2024
Question:
What is digital lending? How digital lending is accelerating the transformation of financial services in India?
 

Introduction:

The introduction of the answer is essential and should be restricted to 3-5 lines. Remember, a one-liner is not a standard introduction

Digital lending represents a dynamic landscape where unconventional financial service providers continuously leverage technological advancements to gain an edge over traditional counterparts. These lenders adeptly integrate digital solutions with simplicity, speed, and comprehensive customer service to streamline loan processes, delivering an end-to-end experience.

Body:

It is the central part of the answer and one should understand the demand of the question to provide rich content

Digital lending is propelling the transformation of financial services in India through various means:

Personalized Communication:

Tech-driven digital lenders harness vast data repositories to analyze consumer behavior and implement hyper-personalization strategies. By segmenting their customer base into distinct subsets based on demographics, location, spending patterns, interaction history, and satisfaction levels, businesses identify target groups more effectively, tailoring their services accordingly.

Enhanced Customer Engagement:

Fostering customer loyalty is pivotal for long-term growth. Consequently, lenders prioritize individualized interactions, viewing customers as unique entities rather than mere statistics. By engaging extensively with their target audience, lenders gather insights to refine their offerings and communication, fostering relationships built on trust and mutual benefit.

Proactive Automation in Credit Decisions:

Traditionally, lending processes were labor-intensive and paper-based. However, digital lenders revolutionize this landscape by integrating fundamental technologies such as Optical Character Recognition (OCR), Robotic Process Automation (RPA), Machine Learning (ML), and Automated Document Recognition (ADR). This proactive automation streamlines lending operations and enhances efficiency.

Emphasis on Cybersecurity and Fraud Detection:

Machine Learning techniques empower financial institutions to detect intricate fraud patterns that elude traditional methods. By maintaining real-time fraud detection systems, these lenders continuously evolve their security measures, ensuring robust protection against fraudulent activities.

Effective Software-as-a-Service (SaaS) Delivery Models:

SaaS delivery models offer agile automation across key lending domains, facilitating rapid deployment, even for complex services like mortgages. This scalable approach not only accommodates diverse borrowing needs but also reduces system costs significantly, enabling resource allocation towards high-value consumer engagements

Conclusion:

The ending of the answer should be on a positive note and it should have a forward-looking approach

In conclusion, the evolution of digital lending is reshaping the financial services landscape in India. By leveraging technology-driven solutions, digital lenders are enhancing customer experiences through personalized communication, proactive automation, and robust cybersecurity measures. This transformation underscores the shift towards a more efficient, customer-centric approach, where the focus is on delivering seamless, end-to-end services tailored to individual needs. As digital lending continues to evolve, it promises to drive further innovation, efficiency, and inclusivity in the financial sector, ultimately contributing to the growth and development of the Indian economy

 

Other Points to Consider

SaaS (Software-as-a-service)

Cybersecurity and Fraud Detection

 

 

Previous Year Questions

What is the status of digitalization in the Indian economy? Examine the problems faced in this regard and suggest improvements. (2023)

Is inclusive growth possible under market economy? State the significance of financial inclusion in achieving economic growth in India. (2022)

(mains - general-studies-3 ) 04-Apr 2024
Question:
What is Multiple Independently Targetable Re-entry Vehicle (MIRV) technology? What makes MIRV technology lethal?
 

Introduction:

The introduction of the answer is essential and should be restricted to 3-5 lines. Remember, a one-liner is not a standard introduction.

The MIRV (Multiple Independently Targetable Re-entry Vehicle) technology enables a single missile to target multiple locations that may be dispersed by hundreds of kilometers.

The latest variant of the Agni-5 missile incorporates MIRV technology, joining the league of countries possessing such capabilities, which includes the United States, Russia, China, France, and the United Kingdom.

Body:

It is the central part of the answer and one should understand the demand of the question to provide rich content

MIRV technology possesses unique lethality due to several factors:

  • Unlike traditional missiles, which carry a single warhead, MIRVs have the capacity to carry multiple warheads simultaneously.

  • These warheads can be released from the missile at varying speeds and trajectories, thereby enhancing their effectiveness in hitting multiple targets.

  • Developing MIRV technology entails significant challenges, including the requirement for large missiles, compact warheads, precise guidance systems, and sophisticated mechanisms for releasing warheads during flight.

Conclusion:

The ending of the answer should be on a positive note and it should have a forward-looking approach

The capability of MIRV technology, allowing a single missile to carry multiple warheads, remains limited to a select few nations—namely, the United States, Russia, China, the United Kingdom, and France.

India's recent successful test of an advanced Agni-5 ballistic missile equipped with MIRV capabilities signifies a significant upgrade in its missile arsenal. This development substantially enhances India's strike capabilities, including its nuclear deterrence potential

 

Other Points to Ponder

DRDO

Other Missiles of DRDO

Agni-5

 

Previous Year Questions

1.How is S-400 air defence system technically superior to any other system presently available in the world? (2021)

2.Discuss India’s achievements in the field of Space Science and Technology. How the application of this technology has helped India in its socio-economic development? (2016)

(mains - general-studies-3 ) 04-Apr 2024
Question:

Why southern India’s water supplies are rapidly running out? Discuss with examples.

 

Introduction:

Southern India is facing a rapid depletion of water supplies due to various factors leading to water scarcity in both rural and urban areas. This situation has serious implications for irrigation, drinking water availability, and agricultural practices.

 

Body:

Reservoir Water Levels: The Central Water Commission (CWC) monitors water levels in key reservoirs across southern states like Andhra Pradesh, Telangana, Karnataka, Kerala, and Tamil Nadu. Recent reports indicate that water stocks in these reservoirs are significantly lower compared to previous years. For instance, reservoirs in Karnataka are currently holding water at only 26% of their full capacity, which is considerably lower than expected levels for this time of the year.

Monsoon Variability: Southern India heavily relies on the monsoon season for replenishing water sources. However, recent years have seen irregular monsoon patterns and inter-seasonal rainfall variability, leading to inadequate recharge of groundwater and reservoirs. Despite the country as a whole receiving normal rainfall, the south peninsular region experienced below-normal rainfall during critical periods of the monsoon, impacting water availability.

Drinking Water Crisis in Urban Areas: Cities like Bengaluru in Karnataka are facing acute drinking water shortages due to depleted water reserves. This crisis extends beyond Bengaluru and affects other urban areas in southern states as well. Urbanization, population growth, and inefficient water management contribute to the strain on water resources in urban centres.

Impact on Irrigated Farming: Southern states, particularly Andhra Pradesh and Tamil Nadu, heavily rely on irrigated farming practices, including high-water-intensive crops like paddy. Water scarcity directly affects agricultural productivity and livelihoods. Diminishing water supplies also leads to conflicts over water allocation among different sectors like agriculture, industry, and domestic usage.

Aquifer Characteristics: Southern India's rocky aquifer system poses challenges as these aquifers do not hold substantial water and deplete quickly. Additionally, the recharge rates may not keep up with extraction rates, exacerbating water scarcity issues.

 

Conclusion:

The water crisis in southern India is a complex issue influenced by climatic factors, unsustainable water management practices, population pressures, and the region's unique hydrological characteristics. Addressing these challenges requires sustainable water conservation measures, efficient irrigation practices, promoting rainwater harvesting, and enhancing water governance to ensure water security for both present and future generations. Government policies, public awareness, and community participation are crucial in mitigating the impacts of water scarcity in southern India.

 

Other Related Topics

Rate of depletion of groundwater in India
How is India addressing its water needs?
Water shortage in Bengaluru
 

 

Previous Year Questions

1. Drought has been recognized as a disaster in view of its spatial expanse, temporal duration, slow onset and lasting effects on vulnerable sections. With a focus on the September 2010 guidelines from the National Disaster Management Authority (NDMA), discuss the mechanisms for preparedness to deal with likely El Niño and La Niña fallouts in India. (2014)

2. Dam failures are always catastrophic, especially on the downstream side, resulting in a colossal loss of life and property. Analyze the various causes of dam failures. Give two examples of large dam failures. (2023)

 

(mains - general-studies-3 ) 02-Apr 2024
Question:
What are the effects of crop residue burning? Discuss how baler machines are helpful in managing stubble.
 
 
Introduction:
 
A Simple Introduction regarding Crop residue burning

Crop residue burning refers to the practice of intentionally setting fire to the leftover plant material (crop residues) after harvesting crops such as rice, wheat, sugarcane, and other agricultural produce. Crop residue burning has significant negative effects on the environment, soil health, and air quality. It leads to the loss of nutrients, impacts soil properties, and contributes to the emission of greenhouse gases and other harmful pollutants. However, the use of baler machines can be a sustainable solution for managing stubble effectively.

 

Body:

The following information in the body of your answer might be appropriate

Effects of Crop Residue Burning

  • Loss of Nutrients: Burning crop residues results in the loss of essential nutrients like nitrogen (N), phosphorus (P), potassium (K), sulfur (S), and organic carbon. This nutrient loss can adversely affect soil fertility and crop productivity.
  • Impact on Soil Properties: The heat generated during residue burning can elevate soil temperatures, leading to the death of beneficial soil organisms. It can also alter soil structure and reduce water retention capacity.
  • Emission of Greenhouse Gases: Crop residue burning releases greenhouse gases (GHGs) such as carbon dioxide (CO2), methane (CH4), and nitrous oxide (N2O), contributing to climate change and air pollution. It also emits other pollutants like carbon monoxide (CO), nitrogen oxides (NOx), and particulate matter (PM), which have adverse health effects.

Baler Machines in Stubble Management

  • Compression of Residues: Baler machines are used to compress agricultural residues such as straw and stubble into compact bales. These bales are easier to handle, transport, and store compared to loose residues.
  • Additional Income Source: Farmers can sell compacted straw bales to industries for various purposes like biofuel production, animal bedding, packaging material, and composting. This provides an additional source of income for farmers.
  • Timely Field Preparation: Using balers allows farmers to clear fields of stubble quickly, enabling timely land preparation and sowing of the next crop without the need for burning.
  • Environmental Benefits: By reducing the practice of crop residue burning, baler machines contribute to improving air quality, preserving soil health, conserving nutrients, and mitigating greenhouse gas emissions.
 
Conclusion:

A Balanced answer in your conclusion should be there 

Baler machines play a crucial role in sustainable agriculture by facilitating effective stubble management and reducing the harmful effects of crop residue burning. Encouraging the adoption of baler technology among farmers, along with promoting alternative uses of stubble, can contribute significantly to environmental conservation and agricultural sustainability. Government initiatives such as subsidies and awareness programs can further promote the widespread adoption of baler machines and sustainable crop residue management practices.

 
Other Related Topics
 
Straw Management System
Happy Seeders
Crop Residue Management (CRM) scheme
 

 

Previous Year Questions

1. Each year a large amount of plant material, cellulose, is deposited on the surface of Planet Earth. What are the natural processes this cellulose undergoes before yielding carbon dioxide, water and other end products? (2022)
2. What are the key features of the National Clean Air Programme (NCAP) initiated by the government of India? (2020)

 

(mains - general-studies-3 ) 04-Apr 2024
Question:

Discuss the initiatives taken by India in adopting solar energy.

 

Introduction:

A Simple introduction about Solar Energy

Solar energy is a renewable and sustainable source of energy derived from the sun's radiation. It is harnessed using various technologies such as photovoltaic (PV) cells, solar panels, concentrated solar power (CSP) systems, and solar thermal collectors. Solar energy has gained significant attention and adoption globally due to its environmental benefits, including reduced greenhouse gas emissions and decreased reliance on fossil fuels. India has been actively pursuing initiatives to harness its vast solar energy potential, recognizing its importance in addressing energy needs, promoting sustainability, and reducing carbon emissions.

Body:

The following information in the body of your answer might be appropriate

Some of the key initiatives taken by India in adopting solar energy include:

Development of Solar Parks and Ultra Mega Solar Power Projects

  • Launched in December 2014, this initiative aims to expedite the setup of solar projects by providing dedicated solar parks.
  • Solar parks are developed in collaboration with state governments, agencies, CPSUs (Central Public Sector Undertakings), and private entrepreneurs.
  • Notable examples include the NTPC Floating Solar Plants in Ramagundam (100 MW), Kayamkulam (92 MW), and the Rihand Dam Floating Solar Power Plant (50 MW).

Grid-Connected Solar Rooftop Programme:

  • The objective is to achieve a cumulative installed capacity of 40,000 MW from grid-connected rooftop solar projects.
  • This initiative promotes the adoption of solar energy in urban and rural areas through rooftop installations.

Pradhan Mantri Kisan Urja Suraksha evam Utthaan Mahabhiyan (KUSUM) scheme:

  • This scheme has three components aimed at promoting renewable energy in agriculture.
  • Component-A focuses on setting up 10,000 MW of decentralized grid-connected renewable energy power plants on barren land.
  • Component B involves the installation of 17.50 lakh stand-alone solar agriculture pumps.
  • Component-C is dedicated to solarizing 10 lakh grid-connected agriculture pumps.

One Sun One World One Grid (OSOWOG):

  • India collaborated with the United Kingdom on this initiative, combining the Green Grids Initiative and OSOWOG.
  • The goal is to launch GGI-OSOWOG at international platforms like the COP26 summit to promote global solar power integration.

Conclusion:

A Balanced answer in your conclusion should be there 

India's proactive measures in adopting solar energy are evident through these initiatives. These efforts not only contribute to clean energy generation but also align with global sustainability goals. With ambitious targets and innovative schemes, India is making significant strides towards a greener and more energy-efficient future.

 

Other Related Topics

Limitations in Installing
Key Components of Solar Energy

 

Previous Year Questions

1. Explain the purpose of the Green Grid Initiative launched at World Leaders Summit of the COP26 UN Climate Change Conference in Glasgow in November, 2021. When was this idea first floated in the International Solar Alliance (ISA)? (2021)

2. India has immense potential of solar energy though there are regional variations in its developments. Elaborate. (2019)

 

(mains - general-studies-3 ) 04-Apr 2024
Question:

What do you mean by dryland agriculture? List the issues related with dryland agriculture.

 

Introduction:

A Simple Introduction about dryland agriculture

Dryland agriculture refers to farming practices conducted in regions characterized by limited soil moisture and high rates of evapotranspiration, resulting in a continuous water deficit throughout the year. In India, approximately 65 per cent of the arable land, totalling around 140 million hectares, falls under rainfed or dryland agriculture. Despite its prevalence, dryland farming faces several challenges that impact productivity and sustainability.

 

Body:

The following information in the body of your answer might be appropriate

Issues associated with dryland agriculture include:

Soil and Moisture Management

  • Variability in soil types, with alfisols and vertisols being predominant in semi-arid regions, while inceptisols and entisols are found in river basins and desert areas.
  • Crops grown in alfisols are highly susceptible to drought stress, whereas vertisols possess better water-holding capacity, reducing drought vulnerability.
  • Water erosion poses a significant threat in mountainous and undulating terrains of Central India.
  • Salinization contributes to soil degradation, particularly in areas affected by irrigation.

Environmental Changes: Waterlogging and Salinity

  • Waterlogging and salinity exacerbate soil degradation issues, often stemming from overirrigation, poor drainage systems, and improper irrigation practices.
  • These problems adversely affect dryland crops, leading to reduced yields and economic losses.
Dietary and Nutritional Considerations
  • Dryland agriculture primarily yields oilseed pulses and coarse grains such as bajra, limiting dietary diversity.
  • The profitability of dryland crops is often low, leading to economic imbalances for farmers and potentially discouraging investment in dryland agriculture.
  • Crop substitution may offer alternatives but requires careful consideration of market demand and ecological suitability.
Conclusion:

A Balanced answer in your conclusion should be there 

Addressing the challenges of dryland agriculture requires holistic approaches that encompass soil and moisture management, environmental sustainability, and diversification of crops. Watershed-based strategies show promise for conserving water and soil resources. Additionally, the adoption of technologies tailored for dryland farming can mitigate climate change impacts and enhance resilience, ultimately contributing to food security and economic well-being in dryland regions.

 

Other Related Topics

Contribution of Drylands to Indian Agriculture

Importance of dry land farming

 

Previous Year Questions
 
1. What is Integrated Farming System? How is it helpful to small and marginal farmers in India? (2022)

 

(mains - general-studies-3 ) 04-Apr 2024
Question:
Describe the blue economy and explain its significance. Does India have a blue economy policy?
 
 

Introduction:

The term "blue economy" refers simply to economic activities linked to the sea and coasts, commonly assumed to incorporate sustainability principles. As defined by the European Commission, it encompasses all economic endeavors related to oceans, seas, and coasts, spanning various established and emerging sectors. The World Bank interprets the blue economy as the sustainable utilization of ocean resources to foster economic growth, enhance livelihoods, and generate employment while preserving the health of the ocean ecosystem.

Body:

The significance of the blue economy is particularly pronounced for a country like India, characterized by an extensive coastline, diverse marine life, and numerous tourism prospects. Finance Minister Sitharaman emphasized the launch of a comprehensive scheme encompassing restoration, adaptation measures, coastal aquaculture, and mariculture, employing an integrated and multi-sectoral approach. This approach ensures economic activities do not compromise the health of the oceans.

Examining India's blue economy policy, a draft policy framework was initially unveiled in July 2022. The policy document outlined key recommendations covering the National Accounting Framework for Blue Economy, Ocean Governance, Coastal Marine Spatial Planning, Tourism Priority, Marine Fisheries, Aquaculture, Fish Processing, Manufacturing, emerging industries, trade, technology, services, skill development, logistics, infrastructure, shipping, coastal and deep-sea mining, offshore energy, security, strategic dimensions, and international engagement.

Conclusion:

In a recent announcement, the Finance Minister highlighted the establishment of five integrated aquaparks and the expansion of the Pradhan Mantri Matsya Sampada Yojana (PMMSY). The expansion aims to increase aquaculture productivity, double exports to Rs 1 lakh crore, and generate 55 lakh employment opportunities in the near future. These initiatives underscore India's commitment to leveraging its blue economy potential for sustainable economic growth and environmental preservation

 

Other Related Topics

 

Coastal aquaculture

Mariculture

 

 

Previous Year Questions

 

Explain the causes and effects of coastal erosion in India. What are the available coastal management techniques for combating the hazard? (2022)

Explain various types of revolutions, took place in Agriculture after Independence in India. How these revolutions have helped in poverty alleviation and food security in India? (2017)

(mains - general-studies-3 ) 16-Feb 2024
Question:
What is nano di-ammonium phosphate (Nano DAP)? Why does the government want to expand its use?
 

Introduction:

Di-ammonium phosphate, commonly known as DAP, stands as the second most widely utilized fertilizer in India following urea. Recognized for its high phosphorus content, crucial for fostering root establishment and development, DAP plays a vital role in enabling plants to achieve their normal size and timely maturity. Nano DAP, produced by the Indian Farmers Fertiliser Cooperative (IFFCO), differs from conventional DAP in its liquid form, containing 8% Nitrogen and 16% Phosphorus by volume.

Body:

Why is the government inclined to promote its usage?

Nano DAP's distinct advantage lies in its tiny particle size, enhancing its efficiency compared to the conventional granular form. This characteristic enables the fertilizer to permeate the seed surface easily or enter through stomata and other plant openings. The improved assimilation within the plant system results in higher seed vigor, increased chlorophyll, enhanced photosynthetic efficiency, improved crop quality, and a subsequent boost in crop yields.

Given the significant subsidies provided by the government on DAP, the adoption of the more cost-effective Nano DAP is expected to alleviate the government's subsidy burden. Its pocket-friendly nature and enhanced convenience contribute to its appeal.

Benefits:

  1. Cost-effectiveness: Nano DAP proves to be more economical.
  2. Convenience: Nano DAP is notably more user-friendly.

Conclusion:

Crucially, India currently relies heavily on fertilizer imports to meet domestic demand. The utilization of domestically produced Nano DAP from Kalol, Gujarat, is anticipated to substantially reduce this import burden. This revolutionary step not only propels Indian agriculture forward in foodgrain production but also establishes self-reliance in fertilizer production. The adoption of Nano DAP signifies a significant stride towards achieving self-sufficiency in fertilizers, providing substantial benefits to Indian farmers

 

Other Related Topics

IFFCO

Nano DAP

 

 

Previous Year Questions

How do subsidies affect the cropping pattern, crop diversity and economy of farmers? What is the significance of crop insurance, minimum support price and food processing for small and marginal farmers? (2017)

(mains - general-studies-3 ) 16-Feb 2024
Question:
What is Multidimensional poverty in India? How Indian Multidimensional Poverty Index is different from global Multidimensional Poverty Index?
 

Introduction:

Traditionally, the measurement of poverty has relied on income levels or, in the absence of income data, expenditure levels. These indicators, known as "poverty lines," represent expenditure levels considered low enough to categorize an individual as impoverished. In India, multidimensional poverty witnessed a decline from 29.17% in 2013-14 to 11.28% in 2022-23, leading to approximately 24.82 crore individuals escaping poverty during this period. Notably, Uttar Pradesh emerged as the state with the highest number of people escaping poverty at 5.94 crore, followed by Bihar at 3.77 crore and Madhya Pradesh at 2.30 crore.

Body:

A divergence exists between the Indian Multidimensional Poverty Index (MPI) and the Global MPI. Globally, the MPI employs 10 indicators across three primary domains: (i) health, (ii) education, and (iii) standard of living. Each of these dimensions holds equal weight, contributing one-third to the final index. Health indicators encompass child and adolescent mortality, education metrics include years of schooling and attendance, and the standard of living dimension incorporates household-specific indicators like housing, household assets, cooking fuel type, sanitation, drinking water, and electricity.

The Indian MPI, while aligning with global dimensions, incorporates two additional indicators to reflect national priorities. These additional indicators cover maternal health under the health dimension and bank accounts under the standard of living dimension, according to the NITI Aayog.

Conclusion:

The Multidimensional Poverty Index (MPI) value for a given population is the proportion of weighted deprivations faced by individuals experiencing multidimensional poverty divided by the total population. This nuanced approach to measuring poverty, considering multiple dimensions beyond income, provides a comprehensive understanding of the challenges individuals face in different aspects of their lives

 

Other Related Points

 

What is the basis for this assessment?

How is MPI calculated?

 

 

Previous Year Questions

COVID-19 pandemic accelerated class inequalities and poverty in India. Comment. (2020)

Despite implementation of various programmes for eradication of poverty by the government in India, poverty is still existing’. Explain by giving reasons. (2018)

An essential condition to eradicate poverty is to liberate the poor from the process of deprivation.” Substantiate this statement with suitable examples. (2016)

(mains - general-studies-3 ) 16-Feb 2024
Question:
What is the credit system carried out by the Central Board of Secondary Education (CBSE)? What changes has the CBSE subcommittee
 

Introduction:

The objective of creditisation is to establish academic parity between vocational and general education, fostering seamless transition between these two educational systems, as outlined in the National Education Policy (NEP) of 2020. In line with this, the University Grants Commission (UGC), responsible for overseeing higher education, introduced the National Credit Framework (NCrF) in 2022 to operationalize this vision. NCrF serves as a unified credit framework aimed at integrating training and skill development into both school and higher education, with student credits digitally stored in the Academic Bank of Credits and accessible through linked Digilocker accounts.

Body:

The proposed changes recommended by the Central Board of Secondary Education (CBSE) subcommittee underscore the absence of a formalized credit system in the current standard school curriculum. According to the CBSE plan, an academic year would consist of 1,200 notional learning hours, equating to earning 40 credits. These hours encompass both academic learning within the school setting and non-academic or experiential learning outside the school premises.

To implement this, adjustments have been made to the scheme of studies, outlining teaching hours and credits earned for each subject. The committee advocates the inclusion of multidisciplinary and vocational courses alongside existing disciplines. Consequently, to pass final exams in Classes 9 and 10, students must complete ten subjects, comprising three languages and seven core disciplines.

Furthermore, the proposal suggests that at least two of the three mandatory languages should be Indian, providing flexibility in language choices such as Hindi, Sanskrit, and English. For Classes 11 and 12, the Board recommends students study six subjects, encompassing two languages and four subjects with an optional fifth. At least one of the two languages must be Indian.

Conclusion:

The Central Board of Secondary Education (CBSE), the largest national school board in the country, is poised to implement substantial modifications to the academic framework for Classes 9, 10, 11, and 12. This initiative aligns with the creditisation approach advocated by the 2020 National Education Policy (NEP), reflecting a commitment to establishing equivalence between vocational and general education

 

Other Related Points to Consider

National Education Policy

Credit system

National Credit Framework

 

 

Previous Year Questions

Examine the uniqueness of tribal knowledge systems when compared with mainstream knowledge and cultural systems. (2021)

Discuss the main objectives of Population Education and point out the measures to achieve them in India in detail. (2021)

(mains - general-studies-3 ) 16-Feb 2024
Question:
What is geo-spatial intelligence? How geospatial solutions will address National Priorities and Sustainable Development Goals?
Introduction
You may incorporate the following into your answer
The term "millets" refers to small-seeded grains such as sorghum (jowar), pearl millet (bajra), foxtail millet (kangni/Italian millet), little millet (kutki), kodo millet, finger millet (ragi/mandua), proso millet (cheena/common millet), barnyard millet (sawa/sanwa/jhangora), and brown top millet (korale). Among the earliest crops cultivated were millets. These grains have been a dietary staple since as early as 3,000 BC in the Indus Valley, and many of the varieties grown globally today have their origins in India. While native varieties of millets exist in China, Japan, and West Africa, they share a long history of cultivation and consumption across various regions
 
Body
You may incorporate the following into your answer

A recent study, published in April of this year in the Agronomy Journal, highlights the transformation of India's primary pearl millet (bajra) production areas. Between 1998 and 2017, these zones have shifted to encompass 18 districts located across eastern Rajasthan and Haryana. The research was a collaboration between the International Crops Research Institute for the Semi-arid Tropics (ICRISAT) and the Indian Council of Agricultural Research – All India Coordinated Research Project on Pearl Millet (ICAR-AICRP).

This alteration in cultivation zones has been attributed to an increase in rainfall induced by human-driven climate change. In India, pearl millet cultivation areas are traditionally classified based on precipitation patterns and soil types:

(i) Zone A1 – Arid Zone of Rajasthan (less than 400 mm rainfall) (ii) Zone A – Semi-arid regions in north and central India (more than 400 mm rainfall) – Rajasthan, Haryana, Gujarat, and Uttar Pradesh (iii) Zone B – Southern and central western India

The study further divides Zone 'A' into three subzones: 'G', 'AE1', and 'AE2'. Zone 'G' witnessed an average increase in rainfall in Gujarat, while Zone 'AE1' experienced a rise in eastern Rajasthan and Haryana. Additionally, Zone 'AE2' covers 12 districts in Uttar Pradesh and Madhya Pradesh.

Conclusion

In India, millets are predominantly cultivated during the kharif season. As per Agriculture Ministry data for 2018-19, three millet crops—bajra (3.67%), jowar (2.13%), and ragi (0.48%)—comprised approximately 7 percent of the total cultivated area in the country.

Millets are renowned for being nutritional powerhouses. Buckwheat (kuttu) and amaranth (chaulai), categorized as pseudo millets due to their substantial nutritional value, are now recognized as "Nutri Cereals" for production, consumption, and trade.

On March 3, 2021, the United Nations General Assembly (UNGA) ratified a resolution designating 2023 as the International Year of Millets. Proposed by India, this initiative garnered support from 72 countries.

Other Points to Consider

Millets under PDS

Benefits of Millets

Main millets-producing states

 

 

Previous Year Questions

Assess the role of National Horticulture Mission (NHM) in boosting the production, productivity and income of horticulture farms. How far has it succeeded in increasing the income of farmers? (2018)

How has the emphasis on certain crops brought about changes in cropping patterns in recent past? Elaborate the emphasis on millets production and consumption. (2018)

(mains - general-studies-3 ) 22-Nov 2023
Question:
Assess the role of the Arctic Council in promoting sustainable development and environmental protection in the Arctic. (250 words)
 
Introduction
You may incorporate the following in your answer
 
The Arctic Council plays a vital role in promoting sustainable development and environmental protection in the Arctic region. This intergovernmental forum, consisting of eight Arctic states and six indigenous organizations, focuses on addressing key issues related to the Arctic environment, sustainable development, and the well-being of its indigenous communities.
 
Body
You may incorporate the following in your answer

First and foremost, the Arctic Council facilitates scientific cooperation and information-sharing among member states. This collaboration results in a better understanding of the region's environmental challenges, including climate change, biodiversity conservation, and pollution control. It allows for informed policy decisions and the implementation of effective environmental protection measures.

The Council also emphasizes the importance of sustainable development in the Arctic. Through various working groups and initiatives, it encourages responsible economic activities that consider the environmental and social impacts. Sustainable resource management, including fisheries and energy extraction, is a key area of focus, as it ensures that economic growth in the Arctic does not come at the cost of its fragile ecosystems.

Furthermore, the Arctic Council acknowledges the rights and concerns of indigenous communities living in the region. Their traditional knowledge and active participation in the Council's activities contribute to culturally sensitive and environmentally sustainable policies and practices.

Conclusion

A simple and balanced conclusion would be ideal

Arctic Council serves as a significant platform for promoting environmental protection and sustainable development in the Arctic. By fostering international cooperation, scientific research, and indigenous involvement, it plays a critical role in addressing the unique challenges facing this sensitive and rapidly changing region.

(mains - general-studies-3 ) 28-Oct 2023
Question:
What are the possible reasons behind the cyclic transformation of the Sahara Desert into savannah and woodland ecosystems?
 
Introduction

A simple introduction to this answer can revolve around a general statement about Sahara Desert.

The Sahara desert covers an area of around 9,200,000 square kilometres and is one of the largest hot deserts in the world.

Deserts are defined by low average annual rainfall — usually 100 millimeters (less than 4 inches) of rain per year or less.

The cyclic transformation of the Sahara Desert into savannah and woodland ecosystems is one of the most remarkable environmental changes on the planet.

According to the research study that was published in the journal Nature Communications, the periodic wet phases in the Sahara desert were caused by changes in Earth’s orbit around the Sun

Body

You may incorporate some of the following points in the body of your answer:

Possible Reasons

Increase in the African Humid period which could have allowed the dispersal of various species.

There is already a lot of evidence to back up the hypothesis that the Sahara was once vegetation. These “North African humid periods” may have had a significant influence in the establishment of vegetation passageways out of Africa.

The greenings of the Sahara Desert could have been driven by changes in Earth’s orbital precession.

The Earth’s spin influences seasonality over an approximately 21,000-year cycle. The changes in precession also decide the amount of energy received by the planet during different seasons.

These changes caused warmer summers in the Northern hemisphere and thereby increased the strength of the West African monsoon system which has increased rain in the Sahara, causing the spread of savannah-type vegetation through the desert.

Conclusion

You can add a simple conclusion like the following

Sahara's transformation is a topic of ongoing research, and our understanding of the mechanisms involved continues to evolve. Additionally, human-induced climate change may be influencing the Sahara's climate in ways that are not fully understood. Studying past climate records, modeling future scenarios, and monitoring current conditions are essential for gaining a more comprehensive understanding of the complex processes involved in the transformation of the Sahara Desert into savannah and woodland ecosystems.

Other Points to Consider

Species living in the Sahara Desert

Countries covered under the Sahara Desert

Rivers flowing through the Sahara Desert

 

Previous Year Questions

1.The process of desertification does not have climate boundaries. Justify with examples. (UPSC CSE Mains, GS3 2020)

2.Major hot deserts in northern hemisphere are located between 20-30 deg N latitudes and on the western side of the continents. Why? (UPSC CSE Mains, GS3 2013)

 

 

(mains - general-studies-3 ) 20-Sep 2023
Question:
The ozone layer is a shield of gas that protects our planet from harmful parts of the Sun’s radiation. Discuss how Montreal Protocol had helped in the reduction of ozone-depleting substances
 
Introduction

A simple introduction to this answer can revolve around a general statement about Ozone.

Ozone is a molecule that is made up of three oxygen atoms. A layer of this gas sits in our planet’s stratosphere between 15 and 30 kilometres above the surface. It absorbs a portion of the radiation from the Sun, preventing it from reaching the planet.

Ozone is a molecule that is made up of three oxygen atoms. A layer of this gas sits in our planet’s stratosphere between 15 and 30 kilometres above the surface.

It absorbs a portion of the radiation from the Sun, preventing it from reaching the planet. It also prevents UV-B radiation from reaching the Earth and harming humans and other living beings

Body

You may incorporate some of the following points in the body of your answer:

Montreal Protocol

  • The 1989 Montreal Protocol is meant to protect the ozone layer of the upper atmosphere.
  • A set of chemicals, mainly the chlorofluorocarbons or CFCs, which were being used in the air-conditioning and refrigeration industry earlier, were found to be damaging the ozone layer of the upper atmosphere.
  • The extensive use had led to depletion of the ozone layer, and formation of an “ozone hole” over the Antarctic region.
  • Montreal Protocol mandated the complete phase-out of CFCs and other ozone-depleting substances (ODS), which it has successfully managed to do in the last three decades.
  • CFCs were gradually phased out, first by HCFCs, or hydrochlorofluorocarbons in some situations, and then by HFCs, which have little ozone-depleting potential.

The Kigali Amendment

  • In 2016, countries agreed to include HFCs in the list of controlled substances under Montreal Protocol and decided on a schedule for its phase-down.
  • If implemented successfully, the Kigali Amendment is expected to prevent about 0.5°C rise in global warming by the end of this century.
Conclusion

Your conclusion should be short. You may include:

  • September 16 is celebrated as Ozone Day. This year it was celebrated under the theme “Montreal Protocol: Fixing the ozone layer and reducing climate change.”
  • The Montreal Protocol is considered the most successful international environmental regulation, having managed to eliminate almost 90 per cent of the ozone-depleting chemicals like chlorofluorocarbons (CFCs) which were widely used in the furniture and air-conditioning industry. Montreal Protocol has a fairly good track record on ensuring climate benefits as well.
  • The UNEP estimates that, with Kigali Amendment, the avoided emissions could touch 420 billion tonnes of carbon dioxide equivalent by the end of the century.
  • India had played a key role in negotiating the Kigali Amendment. It had fought hard to get an extended timeline for itself, and some other countries, for the reduction of HFC use.
  • The 20-year ‘India Cooling Action Plan’, or ICAP, released in 2019, describes cooling as a “developmental need” and seeks to address the rising demand in cooling, from buildings to transport to cold-chains, through sustainable actions.
 
Other Points to Consider

Methane

Greenhouse Gases

Global Warming

 

 

Previous Year Questions

1. Discuss global warming and mention its effects on the global climate. Explain the control measures to bring down the level of greenhouse gases which cause global warming, in the light of the Kyoto Protocol, 1997. (UPSC CSE Mains GS32022)

(mains - general-studies-3 ) 20-Sep 2023
Question:
What is the dual control structure of the Assam Rifles? Discuss the roles and responsibilities carried out by the Assam Rifles
 
Introduction

A simple introduction to this answer can revolve around a general statement about Assam Rifles.

Assam Rifles is one of the six central armed police forces (CAPFs) under the administrative control of the Ministry of Home Affairs (MHA).

The other forces being the Central Reserve Police Force (CRPF), the Border Security Force (BSF), the Indo-Tibetan Border Police (ITBP), the Central Industrial Security Force (CISF) and the Sashastra Seema Bal (SSB).

Assam Rifles is tasked with the maintenance of law and order in the North East along with the Indian Army and also guards the Indo-Myanmar border in the region.

 

Body

You may incorporate some of the following points in the body of your answer:

Dual control structure of the Assam Rifles

  • It is the only paramilitary force with a dual control structure. While the administrative control of the force is with the MHA, its operational control is with the Indian Army, which is under the Ministry of Defence (MoD).
  • It means that salaries and infrastructure for the force is provided by the MHA, but the deployment, posting, transfer and deputation of the personnel is decided by the Army.
  • All its senior ranks, from DG to IG and sector headquarters are manned by officers from the Army. The force is commanded by Lieutenant General from the Indian Army.
  • Its recruitment, perks, promotion of its personnel and retirement policies are governed according to the rules framed by the MHA for CAPFs.
  • This has created two sets of demands from both within the Assam Rifles and by MoD and MHA for singular control over the force by one ministry.

Role and Contribution

  • Assam Rifles is the oldest paramilitary force raised way back in 1835 in British India with just 750 men. Since then, it has gone on to fight in two World Wars, the Sino-Indian war of 1962 and used as an anti-insurgency force against militant groups in the North East.
  • It was reorganised later as Assam Frontier Force as its role was expanded to conduct punitive operations beyond Assam borders.
  • The Post-Independence role of the Assam Rifles continued to evolve ranging from conventional combat role during Sino-India War 1962, operating in foreign land as part of the Indian Peace Keeping Force (IPKF) to Sri Lanka in 1987 (Op Pawan) to peacekeeping role in the North-Eastern areas of India.
  • It remains the most awarded paramilitary force in both pre- and post-independent India.
  • Assam Rifle is responsible for the maintenance of law and order in the North East along with the Indian Army and also guards the Indo-Myanmar border in the region.
Conclusion

Your conclusion should be short. You may include:

Today the Force remains deployed in some of the most remote and under developed areas and provides security to locals. Assam Rifles has grown substantially over the years from 17 battalions in 1960 to 46 battalions at present.

The Force also has a Training Centre and a number of Logistics Units. Through its long deployment in the tribal belt, the Assam Rifles has earned the complete confidence of the locals and has helped considerably in bringing the people of this region into the national main stream.

Other Points to Consider

What is happening with the Assam Rifles in Manipur?

Why do both MHA and MoD want full control over Assam Rifles?

 

Previous Year Questions

1.The North-Eastern region of India has been infested with insurgency for a very long time. Analyze the major reasons for the survival of armed insurgency in this region. (2017)

(mains - general-studies-3 ) 12-Sep 2023
Question:
What is the significance of ozone? Discuss the reasons behind the ozone hole.
 
Introduction
A simple introduction to this answer can revolve around a general statement about Ozone.

Ozone (chemically, a molecule of three oxygen atoms) is found mainly in the upper atmosphere, an area called stratosphere, between 10 and 50 km from the earth’s surface.

Ozone is present in the atmosphere in low concentrations and at places where this layer is thickest, there are not more than a few molecules of ozone for every million air molecules.

Why it is important?

The ozone layer protects life on Earth from the sun’s harmful ultraviolet (UV) rays.

The ozone molecules eliminate a big threat to life forms on Earth. UV rays can cause skin cancer and other diseases and deformities, in plants and animals

Body

You may incorporate some of the following points in the body of your answer:

  • During experiments in Antarctica in the early 1980s, scientists noticed that during September-November, the concentration of ozone fell considerably lower than what was recorded in the 1950s.
  • The destruction of the ozone layer became a major international issue in 1985 when the “ozone hole” was discovered over Antarctica.
  • The ‘ozone hole’ is not really a hole. It is a region in the stratosphere, directly above Antarctica, where the concentration of ozone has been measured to become extremely low in certain months.

Causes

  • The depletion of the ozone layer is not limited to that area and has happened in other regions of the stratosphere as well, but a set of special meteorological and chemical conditions that arise over Antarctica in the months of September, October and November.
  • The 1987 Montreal Protocol on Substances That Deplete the Ozone Layer (the only universally ratified treaty in United Nations’ history), began the phase-out of CFCs in 1993.
  • By 2005 the consumption of ozone-depleting chemicals controlled by the agreement had fallen by 90–95 percent in the 197 countries that were parties to the protocol.
  • There are 96 chemicals are presently controlled by the Montreal Protocol, including:
  • Halo-carbons (Chlorofluorocarbons (CFCs) and Halons): These were considered wonder gases because they are long-lived, non-toxic, non-corrosive, and non-flammable.
  • CFC-11 remains in the atmosphere for 50 years, CFC-12 for 102 years, and CFC-115 for 1,700.
  • Halon 1301 is used primarily in fire extinguishers and has an atmospheric lifetime of 65 years.
  • Carbon tetrachloride: It is used as a solvent and takes about 42 years to break down in the atmosphere.
  • Hydrochlorofluorocarbons (HCFCs): These were developed as the first major replacement for CFCs. While much less destructive than CFCs, HCFCs also contribute to ozone depletion. They have an atmospheric lifetime of about 1.4 to 19.5 years.
  • There are various other chemicals that can be mentioned like Methyl chloroform, Hydrobromofluorocarbons (HBFCs), Methyl bromide, Bromochloromethane (BCM), etc.
Conclusion

Your conclusion should be short. You may include:

The elimination of ozone-depleting substances has an important climate change co-benefit as well.

These substances also happen to be powerful greenhouse gases, several of them hundreds or even thousands of times more dangerous than carbon dioxide, the most abundant greenhouse gas and the main driver of global warming.

 

Other Points to Consider

Kigali Agreement

India’s effort

 

Previous Year Questions

1.Troposphere is a very significant atmosphere layer that determines weather processes. How? (2022)

 

(mains - general-studies-3 ) 12-Sep 2023
Question:
Methane is the primary component of natural gas and is a potent greenhouse gas (GHG) with a global warming potential (GWP). How we can reduce methane emissions?
 
Introduction

A simple introduction to this answer can revolve around a general statement about Methane.

Methane is a greenhouse gas, which is responsible for 30 per cent of the warming since preindustrial times, second only to carbon dioxide. A report by the United Nations Environment Programme observed that over a 20-year period, methane is 80 times more potent at warming than carbon dioxide.

There are various sources of methane including human and natural sources. Human sources of methane include landfills, oil and natural gas systems, agricultural activities, coal mining, wastewater treatment, and certain industrial processes

Body

 

You may incorporate some of the following points in the body of your answer:

Measures to Reduce Methane Emissions

India has been persistent in its commitment to developing and implementing climate change policies. The following are ongoing attempts to limit methane emissions:

National Mission on Sustainable Agriculture (NMSA): It involves climate-resilient practices such as methane reduction in rice agriculture which helps to significantly reduce methane emissions.

National Innovations in Climate Resilient Agriculture (NICRA) project: The Indian Council of Agricultural Research (ICAR) has developed several technologies with mitigation potential for methane from rice viz.

(a) System for Rice Intensification: The technique has potential to enhance rice yield from 36-49% with about 22-35% less water than conventional transplanted rice;

(b) Direct Seeded Rice: The system reduces methane emissions as it does not involve raising nurseries, puddling and transplanting. Unlike transplanted paddy cultivation, standing water is not maintained in this system and

(c) Crop Diversification Programme: Methane emissions is avoided due to diversion of paddy to alternate crops like pulses, oilseeds, maize, cotton and agroforestry.

Capacity-building programmes are conducted through Krishi Vigyan Kendras across the country to create awareness on climate-resilient practices.

National Livestock Mission: It includes Breed Improvement and Balanced Rationing. Feeding better quality balanced rations to animals helps to minimise methane emissions from livestock. The Government of India also promotes for green fodder production, silage making, chaff cutting, and total mixed ration.

Gobar (Galvanizing Organic Bio-Agro Resources) –Dhan’ scheme: It supports biodegradable waste recovery and conversion of waste into resources and reduction of methane emissions.

 

Conclusion

 

Your conclusion should be short. You may include:

COP28 President Designate Sultan Ahmed Al Jaber called upon the oil and gas industry to phase out methane emissions by 2030 and align in favour of comprehensive net-zero emission plans by or before 2050.

According to the Paris-based International Energy Agency (IEA), methane emissions are among the top causes of global warming. Fossil fuel operations generate over one-third of all methane emissions from human activity. Thus, action on methane is seen as one of the most effective measures the energy sector can take to mitigate climate change

Other Points to Consider

Global Methane Pledge

Global Methane Initiative

Waste to Energy

 

Previous Year Questions
 
1.Discuss global warming and mention its effects on the global climate. Explain the control measures to bring down the level of greenhouse gases which cause global warming, in the light of the Kyoto Protocol, 1997. (2022)
(mains - general-studies-3 ) 05-Sep 2023
Question:
Aditya L1 is the first Indian space-based mission to study the Sun. What are the payloads of the mission? Discuss the objectives of the Aditya L1
 
Introduction

A simple introduction to this answer can revolve around a general statement about Aditya L1

  • The Indian Space Research Organisation (ISRO) launched Aditya L-1, the first Indian space-based mission to study the Sun, from the Satish Dhawan Space Centre in Sriharikota.
  • The solar probe was carried into space by the Polar Satellite Launch Vehicle (PSLV) in ‘XL’ configuration. SLV is one of the most reliable and versatile workhorse rockets of ISRO.
  • The PSLV will initially place the Aditya L-1 in a lower Earth orbit. Subsequently, the spacecraft’s orbit around the Earth will be raised multiple times before it is put on a path to a halo orbit around the L1 Lagrange point
Body

You may incorporate some of the following points in the body of your answer:

Payloads of the Mission

The spacecraft carries seven scientific payloads for systematic study of the Sun. All payloads are indigenously developed in collaboration with various ISRO Centres.

Visible Emission Line Coronagraph

  • It is designed to study solar corona and dynamics of coronal mass ejections.
  • The payload is developed by the Indian Institute of Astrophysics, Bengaluru in close collaboration with ISRO.
  • VELC is the prime payload onboard Aditya-L1, designed as a reflective coronagraph with a multi-slit spectrograph.
  • Solar Ultra-violet Imaging Telescope (SUIT) to image the Solar Photosphere and Chromosphere in near Ultra-violet (UV) and, to measure the solar irradiance variations in near UV.
  • The payload is developed by Inter-University Centre for Astronomy and Astrophysics, Pune in close collaboration with ISRO.
  • SUIT is a UV telescope to image the solar disk in the near ultra-violet wavelength range.
  • Solar Low Energy X-ray Spectrometer (SoLEXS) and High Energy L1 Orbiting X-ray Spectrometer (HEL1OS) are designed to study the X-ray flares from the Sun over a wide X-ray energy range. Both these payloads are developed at U R Rao Satellite Centre, Bengaluru.
  • SoLEXS is a soft X-ray spectrometer onboard Aditya-L1. The payload is designed to measure the solar soft X-ray flux to study solar flares.
  • Aditya Solar Wind Particle Experiment (ASPEX) and Plasma Analyser Package for Aditya (PAPA) payloads are designed to study the solar wind and energetic ions, as well as their energy distribution.
  • Magnetometer payload is capable of measuring interplanetary magnetic fields at the L1 point. The payload is developed at Laboratory for Electro-Optics Systems, Bengaluru.

Objectives of the Aditya L-1

The main objective of the mission is to expand our knowledge of the Sun, and how its radiation, heat, flow of particles, and magnetic fields affect us. There are various other objectives that the mission will embark upon:

(i) To study the upper atmospheric layers of the Sun called chromosphere and corona. While the corona is the outermost layer, the chromosphere is just below it.

(ii) To examine coronal mass ejections (CMEs), which are large expulsions of plasma and magnetic fields from the Sun’s corona.

(iii) To analyse the corona’s magnetic field and the driver of the space weather.

(iv) To understand why the Sun’s not-so-bright corona is a million degree Celsius hot when the temperature on the surface of the Sun is just about 5,500 degree Celsius.

(v) To help scientists know the reasons behind the acceleration of particles on the Sun, which leads to the solar wind — the constant flow of particles from the Sun.

 

Conclusion

Your conclusion should be short. You may include:

According to ISRO, the Sun “emits radiation/light in nearly all wavelengths along with various energetic particles and magnetic fields. The atmosphere of the Earth as well as its magnetic field acts as a protective shield and blocks a number of harmful wavelength radiations including particles and fields.”

The mission hopes to generate user-friendly information that can help safeguard a range of satellite-dependent operations such as telecommunications, mobile-based Internet services, navigation, power grids, etc.

Among the five Lagrange points, L1 is the most favoured to get an unhindered view of the Sun. L2 is located behind the Earth, and thus obstructs the view of the Sun, while L3 is behind the Sun which is not a great position to communicate with Earth. L4 and L5 are good and stable locations, but are much farther from Earth compared to L1, which is directly between the Sun and the Earth.

 

Other Points to Consider

 

What is Halo Orbit?

What are Lagrange points?

What is space weather?

 

 

Previous Year Questions

1.India has achieved remarkable successes in unmanned space missions including the Chandrayaan and Mars Orbiter Mission, but has not ventured into manned space mission. What are the main obstacles to launching a manned space mission, both in terms of technology and logistics? Examine critically. (2017)

 

(mains - general-studies-3 ) 05-Sep 2023
Question:
The Waste to Wealth mission brings scientific processing of waste to the forefront to build a zero landfill and zero waste nation. Elucidate.
 
Introduction
A Simple Introduction would be enough to address the above Question
 
 Waste to Wealth mission is a national initiative to transform the way we manage our waste. It aims to achieve zero landfills and zero waste by 2030 by promoting scientific processing of waste. This includes recycling, composting, and anaerobic digestion
 
The Waste to Wealth mission is a bold and ambitious goal, but it is achievable. By working together, we can create a cleaner, healthier, and more sustainable future for our country
 
Body

You may incorporate some of the following points in the body of your answer:

The ‘Waste to Wealth Mission’ of the Prime Minister’s Science, Technology, and Innovation Advisory Council (PM-STIAC) aims to identify, develop, and deploy technologies to treat waste to generate energy, recycle materials, and extract resources of value.

The mission is working to identify and support the development of new technologies that promise to create a clean and green environment.

The objective of the mission is:

(i) To identify and support the development of new technologies that can help create a cleaner and greener environment.

(ii) To boost and augment the Swachh Bharat Mission and Smart Cities Project by leveraging science, technology, and innovation.

(iii) To create circular economic models that are financially viable for waste management.

(iv) To streamline waste handling in India.

The mission will assist and augment the Swachh Bharat and Smart Cities projects by leveraging science, technology, and innovation to create circular economic models that are financially viable for waste management to streamline waste handling in the country.

The Waste to Wealth Mission is one of the nine scientific missions of the Prime Minister’s Science, Technology, and Innovation Advisory Council (PMSTIAC).

In addition, the objective will seek out and assist the development of innovative technologies that promise to build a clean and green environment.

The mission will support and supplement the Swachh Bharat and Smart Cities programs by harnessing science, technology, and innovation to develop circular economic models that are commercially feasible for waste management in order to simplify trash management in the country.

You can add the following schemes in your answer

GOBAR-Dhan” (Galvanising Organic Bio-Agro Resources-Dhan) initiative has two objectives:

(i) To make villages clean and generate wealth.

(ii) Energy from cattle and other waste.

Houseboat Sewage Management

The Waste to Wealth Mission will assist the Government of Jammu and Kashmir in piloting, verifying, and scaling up decentralized sewage treatment technology to address the input of untreated sewage from houseboats into Dal Lake.

Decentralized solid and liquid waste management

The Waste to Wealth Mission is engaging with Municipal Corporation Faridabad (MCF) to jointly deploy technologies in solid and liquid waste management to address Faridabad’s enormous waste challenges.

Swachhta Saarthi Fellowship

The fellowship is a program to empower young innovators participating in community waste management/awareness campaigns/waste surveys/studies.

 

Conclusion

 

Your conclusion should be short.

According to the Ministry of Housing and Urban Affairs, about 4 lakh women are directly engaged in sanitation and waste management in cities.

The Waste to Wealth mission acts as a platform for technology providers, government stakeholders, urban local bodies and users to seek an appropriate solution.

 

Other Points to Consider

Challenges

 

 

Previous Year Questions

1.What are the impediments disposing the huge quantities of discarded solid waste which are continuously being generated? How do we remove safely the toxic wastes that have been accumulated in our habitable environment? (2018)

(mains - general-studies-3 ) 23-Aug 2023
Question:
 Discuss how the National Quantum Mission will work towards strengthening India’s research and development in the quantum arena
 
Introduction

A simple introduction to this answer can revolve around a general statement about quantum computers and National Quantum Mission.

A quantum computer seeks to exploit the laws that govern the behaviour of atoms and subatomic particles. In a quantum computer, information is stored in quantum bits, or qubits. And a qubit can be both 0 and 1 at the same time

National Quantum Mission (NQM) will help to scale up scientific and industrial R&D, for accelerating Quantum Technology-led economic growth and leverage India into a leading nation in the area

Body

 

You may incorporate some of the following points in the body of your answer:

  • The NQM focuses on indigenously building quantum-based (physical qubit) computers which are far more powerful and are able to perform the most complex problems in a highly secure manner.
  • It will target developing intermediate-scale quantum computers with 50-1000 physical qubits in eight years in various platforms like superconducting and photonic technology.
  • Communications: Satellite-based secure quantum communications between ground stations over a range of 2000 kilometres within India, Long-distance secure quantum communications with other countries and inter-city quantum key distribution over 2000 km as well as a multi-node Quantum network with quantum memories are among the other objectives of the mission.
  • The mission will help develop magnetometers with high sensitivity in atomic systems and Atomic Clocks for precision timing, communications and navigation.
  • It will also aid in the design and synthesis of quantum materials for quantum device production, such as superconductors, innovative semiconductor architectures, and topological materials.
  • Four ‘Thematic Hubs‘ (T-Hubs) will be established in premier university and national R&D institutes in the fields of quantum computing, quantum communication, quantum sensing and metrology, and quantum materials and devices. The hubs will focus on the development of new knowledge through basic and applied research, as well as the promotion of R&D.
  • Single photon sources/detectors and entangled photon sources will also be developed for quantum communications, sensing and metrological applications
 
Conclusion
(mains - general-studies-3 ) 23-Aug 2023
Question:
India has added significant photovoltaics capacity over the past decade, achieving over 50 GW by 2022. Discuss the recent initiatives taken by India in adopting solar energy.
 
Introduction
 

A simple introduction to this answer can revolve around a general statement about solar energy.

India has enormous solar energy potential. Solar photovoltaic electricity may be successfully harnessed, giving enormous scalability in India. Solar also permits dispersed power generation and quick capacity increase with short lead periods. Off-grid decentralized and low-temperature applications will be helpful for rural applications as well as addressing other energy demands for electricity, heating, and cooling in both rural and urban locations.

Solar energy has had a noticeable influence on the Indian energy landscape in recent years. Millions of people in Indian communities have profited from solar energy-based decentralized and distributed applications that satisfy their cooking, lighting, and other energy demands in an environmentally benign manner.

 

Body

 

You may incorporate some of the following points in the body of your answer:

 

Initiatives taken by India

Development of Solar Parks and Ultra Mega Solar Power Projects

  • It was rolled out in December 2014 with the objective to facilitate the solar project developers to set up projects expeditiously.
  • The solar parks are developed in collaboration with the State Governments and their agencies, CPSUs, and private entrepreneurs.
  • The scheme’s second phase has an objective to set up solar PV projects through Government Producers using domestic cells & modules in WTO compliant manner to facilitate national energy security and environmental sustainability for Government purposes.

Examples: NTPC Floating Solar Plant Ramagundam (100 MW), NTPC Floating Solar Plant Kayamkulam (92 MW), Rihand Dam Floating Solar Power Plant (50 MW), etc.

Grid Connected Solar Rooftop Programme

The objective of the scheme is to achieve a cumulative installed capacity of 40,000 MW from Grid Connected Rooftop Solar (RTS) projects.

Pradhan Mantri Kisan Urja Suraksha evam Utthaan Mahabhiyan (KUSUM) scheme

The scheme has three components:

  • Component-A: For Setting up 10,000 MW of Decentralized Grid Connected Renewable Energy Power Plants on barren land.
  • Component-B: For Installation of 17.50 Lakh stand-alone solar agriculture pumps.
  • Component-C: For Solarisation of 10 Lakh Grid Connected Agriculture Pumps.

One Sun One World One Grid (OSOWOG)

  • The idea for the One Sun One World One Grid (OSOWOG) initiative was put forth by the Hon’ble Prime Minister of India Shri Narendra Modi, at the First Assembly of the International Solar Alliance (ISA) in October 2018.
  • In May 2021, the United Kingdom and India agreed to combine forces of the Green Grids Initiative and the One Sun One World One Grid initiative and jointly launch GGI-OSOWOG at the COP26 summit being hosted by the UK at Glasgow in November 2021.
 
Conclusion

Your conclusion should be short. You may include:

According to the research by Global Energy Monitor, India plans to add 76 gigawatts (GW) of utility-scale solar and wind power by 2025 which could save up to $19.5 billion a year (over 15 lakh crore) caused due to the burning of coal.

India stands 4th in solar PV deployment across the globe as on end of 2021. Solar power installed capacity has reached around 61.97 GW as on November 30, 2022. Presently, solar tariff in India is very competitive and has achieved grid parity.

 

Other Points to Consider

 

Key Components of Solar Energy

Targets declared at COP 26

Limitations in Installing

 

 

 

Previous Year Questions

 

1.Explain the purpose of the Green Grid Initiative launched at World Leaders Summit of the COP26 UN Climate Change Conference in Glasgow in November, 2021. When was this idea first floated in the International Solar Alliance (ISA)? (2021)

2.India has immense potential of solar energy though there are regional variations in its developments. Elaborate. (2019)

(mains - general-studies-3 ) 18-Aug 2023
Question:
James Webb is an international collaboration between NASA, ESA (the European Space Agency), and the Canadian Space Agency (CSA). What are the goals of the mission? How it is different from Hubble Telescope?
 
Introduction

A simple introduction to this answer can revolve around a general statement about james webb telescope.

James Webb Space Telescope (JWST) is an international collaboration between NASA, ESA (the European Space Agency), and the Canadian Space Agency (CSA).

The James Webb Space Telescope (sometimes called JWST or Webb) will be a large infrared telescope with a 6.5-meter primary mirror. The telescope was launched on an Ariane 5 rocket from French Guiana in 2021.

It will explore every stage of our Universe’s history, from the first brilliant glows after the Big Bang to the development of solar systems capable of supporting life on planets such as Earth and the evolution of our own Solar System.

 

Body

You may incorporate some of the following points in the body of your answer:

 

Goals of James Webb Telescope

  • The science goals for the Webb can be grouped into four themes:
  • To look back around 13.5 billion years to see the first stars and galaxies forming out of the darkness of the early universe.
  • To compare the faintest, earliest galaxies to today’s grand spirals and understand how galaxies assemble over billions of years.
  • To see where stars and planetary systems are being born.
  • To observe the atmospheres of extrasolar planets (beyond our solar system), and perhaps find the building blocks of life elsewhere in the universe.

Difference between James Webb and Hubble Telescopes

  • James Webb will primarily look at the Universe in the infrared while Hubble primarily studied optical and ultraviolet wavelengths (though it has some infrared capability).
  • Webb has a much bigger mirror than Hubble which means light collecting area of Webb can peer farther back into time than Hubble is capable of doing.
  • Hubble is in a very close orbit around the earth, while Webb will be 1.5 million kilometers (km) away at the second Lagrange (L2) point.
  • The JWST will be able to see right through and into massive clouds of dust that are opaque to earlier generation visible-light observatories like the Hubble Telescope.
  • Webb is equipped with cameras and other instruments sensitive to infrared or “heat” radiation, and the Hubble is not
 
Conclusion

Your conclusion should be short. You may include:

The JWST is the largest and most powerful telescope ever built. Recently, the United States space research agency NASA said the James Webb Space Telescope (JWST) has produced the deepest and sharpest infrared image of the distant universe, a major event in astronomy.

James Webb Space Telescope will be “a giant leap forward in our quest to understand the Universe and our origins”, as it will examine every phase of cosmic history: from the Big Bang to the formation of galaxies, stars, and planets to the evolution of our own Solar System.

 

 

 

Previous Year Questions

1.Launched on 25th December, 2021, James Webb Space Telescope has been much in the news since then. What are its unique features which make it superior to its predecessor Space Telescopes? What are the key goals of this mission? What potential benefits does it hold for the human race? (2022)

 

(mains - general-studies-3 ) 18-Aug 2023
Question:
 
The year 2023 is the International Year of Millets. Discuss the benefits of including millet as a cereal.

Introduction

A simple introduction to this answer can revolve around a general statement about millets.

The United Nations has declared 2023 as the International Year of Millets. Since that was at the initiative of India, which also accounts for a fifth of the world’s millet production.
India is the largest producer of millet in the world. India’s two varieties of millet namely Pearl Millet (Bajra) and Sorghum (Jowar) together contributed approximately 19 per cent to world production in 2020.
The major millets producing states in India are Rajasthan, Karnataka, Maharashtra, Uttar Pradesh, Haryana, Gujarat, Madhya Pradesh, Tamil Nadu, Andhra Pradesh and Uttarakhand.

Body

You may incorporate some of the following points in the body of your answer:

Benefits of Millet as a Cereal

The protein, dietary fibre, iron and calcium contents in millet are 2-10 times higher than staple cereals including wheat and rice. Therefore, it is always good to consume millet-based foods which are rich in nutrients and have anti-inflammatory and anti-oxidative properties.
Millets are often the only crops that can grow in arid regions, requiring only 300– 400 mm of water compared to 1400–1500 mm for rice cultivation and 1900-2000 mm for sugar cane.
Millets are carbon-neutral crops by carbon absorption from the environment equivalent to their carbon emissions of 0.1–0.2 CO2eq kg per kg of production, compared to 0.4 CO2eq kg per kg of rice production and 0.35 CO2eq kg per kg of wheat production.
Millet is a short-duration crop that is good for a farmer. Millet is known as an efficient crop that takes only 60–90 days to mature while other fine cereals need 100–140 days.

Millets are the hardiest, most resilient, and climate-adaptable crops in harsh, hot (up to 64 degrees Celsius) and drought environments.
Millet contains a lot more nutritional value than rice and wheat which can be able to fulfil our hunger for nutrition.
Millet releases sugar very slowly in comparison with rice and wheat. Sugar dissolves with blood after a long time of eating, so you will not get hungry frequently.

Conclusion

Your conclusion should be short.

Multiple varieties of millets are produced in India such as Pearl Millets, Sorghum, Finger Millet, Foxtail, Kodo, Barnyard, Proso, Little Millet and Pseudo Millets like Buckwheat and Amaranths. Pearl millet (Bajra), Sorghum (Jowar) and Finger Millet (Ragi) constitute the largest share of India’s total production of millets.
The Indian Agricultural Research Institute (IARI) has bred Pusa-1201, a hybrid bajra that gives an average grain yield of over 2.8 tonnes and a potential of 4.5 tonnes per hectare. It matures in 78-80 days and is resistant to downy mildew and blast, both deadly fungal diseases.

Other Points to Consider

1. Government Initiatives towards Millet Cultivation

Previous Year Questions

1. How has the emphasis on certain crops brought about changes in cropping patterns in the recent past? Elaborate the emphasis on millets production and consumption. (2018)
(mains - general-studies-3 ) 11-Aug 2023
Question:
What do you mean by bioenergy, and how is it generated? Discuss how it will aid in meeting India’s energy needs. 

Introduction

A simple introduction to this answer can revolve around a general statement about bioenergy.

Biomass has always been an important energy source for the country and it is renewable, widely available, carbon-neutral and has the potential to provide significant employment in rural areas.

About 32 per cent of the total primary energy use in the country is still derived from biomass and more than 70 per cent of the country’s population depends upon it for its energy needs.
Ministry of New and Renewable Energy has initiated several programmes for the promotion of efficient technologies for their use in various sectors of the economy to ensure the derivation of maximum benefits.
Biomass materials used for power generation include bagasse, rice husk, straw, cotton stalk, coconut shells, soya husk, de-oiled cakes, coffee waste, jute wastes, groundnut shells, sawdust, etc.
For efficient utilization of biomass, bagasse-based cogeneration in sugar mills and biomass power generation have been taken up under the biomass power and cogeneration programme.

Body

You may incorporate some of the following points in the body of your answer:
 
Process

The thermochemical processes for the conversion of biomass to useful products involve combustion, gasification or pyrolysis. The most commonly used route is combustion.
The cycle used is the conventional Rankine cycle with biomass being burnt in a high-pressure boiler to generate steam and operating a turbine with the generated steam.
The exhaust of the steam turbine can either be fully condensed to produce power or used partly or fully for another useful heating activity.
In India, the cogeneration route finds application mainly in industries. The sugar industry has been traditionally practising cogeneration by using bagasse as a fuel.

Aid in meeting India’s energy needs
The two most common biofuels are ethanol and biodiesel:
Ethanol: It is produced by fermentation of residues of crops like corn and sugarcane. The ethanol after fermentation is mixed with petroleum, which dilutes the latter and reduces the emissions. Ethanol-10 or E10 is the most common blend in which 10 per cent composition is Ethanol.
Biodiesel: It is produced from used cooking oil, yellow grease or animal fats. During its production, cooking oil or fat is burned with alcohol in the presence of a catalyst, which produces Biodiesel.
National Policy on Biofuel, implemented in 2018 aims at promoting the production of biofuels under the “Make in India” program.
The main amendments approved to the National Policy on Biofuels:
(i) to allow more feedstocks for the production of biofuels,
(ii) to advance the ethanol blending target of 20 per cent blending of ethanol in petrol to ESY 2025-26 from 2030,
(iii) to promote the production of biofuels in the country, under the Make in India program, by units located in Special Economic Zones (SEZ)/ Export Oriented Units (EoUs),
(iv) to grant permission for the export of biofuels in specific cases.

Conclusion

Your conclusion should be short.

The amendment proposal will pave the way for Make in India drive thereby leading to a reduction in the import of petroleum products by the generation of more and more biofuels.
Over 800 biomass power and bagasse/Non-bagasse cogeneration projects aggregating to 10205.61 MW capacity have been installed in the country for feeding power to the grid.
States which have taken leadership positions in the implementation of bagasse cogeneration projects are Maharashtra, Karnataka, Uttar Pradesh, Tamil Nadu and Andhra Pradesh. The leading States for biomass power projects are Chhattisgarh, Madhya Pradesh, Gujarat, Rajasthan and Tamil Nadu.


Other Points to Consider

1. Limitations of Biofuel
2. Challenges in implementation
(mains - general-studies-3 ) 11-Aug 2023
Question:
What are the different types of cyber-attacks? Discuss steps to strengthen cyber security in India
 
Introduction

A simple introduction to this answer can revolve around a general statement about cyber space and cyber security.

  • Cyberspace is a global domain within the information environment consisting of the interdependent network of information technology infrastructures, including the Internet, telecommunications networks, computer systems, and embedded processors and controllers.
  • Cyber Intrusions and Attacks have increased dramatically over the last decade, exposing sensitive personal and business information, disrupting critical operations, and imposing high costs on the economy.
  • India ranks 3rd in terms of the highest number of internet users in the world after the USA and China which makes it more vulnerable to threats.
  • The Cyber & Information Security (CIS) division of MHA deals with matters relating to cybersecurity, cybercrime, national information security policy and guidelines (NISPG) and its implementation, and the national intelligence grid.

The different types of threats which have been identified are:

Malware – Malicious software to disrupt computers

Viruses and worms

Theft of Intellectual Property or Data

Hacktivism – Cyber protests that are socially or politically motivated

Mobile Devices and applications and their associated Cyber Attacks

Social Engineering – Entice Users to click on malicious links

Spear Phishing – Deceptive Communications (e-mails, texts, tweets)

Domain Name System (DNS) Attacks

Router Security – Border Gateway Protocol (BGP) Hijacking

Denial of Service (DoS) – blocking access to websites

The motives are different for different cyber players such as:

Cybercriminals seek commercial gain from hacking banks and financial institutions as well as phishing scams.

Cyber terrorist seeks to penetrate and attack critical assets and national infrastructure.

Cyber espionage seeks to penetrate both corporate and military data using stealthy IT malware.

Cyber hacktivists have political agendas that hack sites and servers to virally communicate messages.

 

Body

You may incorporate some of the following points in the body of your answer:

Steps to Strengthen Cyber Security in India

1.Network Security: Protect your networks against external and internal attacks.

2.Malware Protection: Produce relevant policies and establish anti-malware defences that are applicable and relevant to all business areas.

3.Incident Management

  • Establish an incident response and disaster recovery capability.
  • Produce and test incident management plans.
  • Provide specialist training to the incident management team.
  • Report criminal incidents to law enforcement.

4.Monitoring: Establish a monitoring strategy and produce supporting policies. Continuously monitor all ICT systems and networks.

5.User Education and Awareness

  • Produce user policies covering acceptable and secure use of the organisation’s systems.
  • Establish a staff training programme.

6.Home and Mobile Working

  • Develop a mobile working policy and train staff to adhere to it.
  • Apply the secure baseline to all devices.

7.Secure Configuration: Apply security patches and ensure that the secure configuration of all ICT systems is maintained.

8.Removable Media Controls: Produce a policy to control all access to removable media.

9.Managing User Privileges: Establish account management processes and limit the number of privileged accounts.

10.Information Risk Management Regime: Establish and effective governance structure and determine your risk appetite.

Conclusion

 

Your conclusion should be short.

The government shall take appropriate steps for enhancing the awareness of citizens and small businesses for cybersecurity.

Cybersecurity Capacity building and training for professionals, extending the ISEA program, introducing curricula academia and organizing conferences.

 

Other Points to Consider

Cyber Security Challenges

 

Previous Year Questions

1.Discuss the potential threats of Cyber attack and the security framework to prevent it. (2017)

2.Discuss different types of cyber crimes and measures required to be taken to fight the menace. (2020)

3.What are the different elements of cyber security? Keeping in view the challenges in cyber security, examine the extent to which India has successfully developed a comprehensive National Cyber Security Strategy. (2022)

(mains - general-studies-3 ) 02-Aug 2023